Download as pdf or txt
Download as pdf or txt
You are on page 1of 84

LIMITS

COMPLETE SET OF MODULES


Prerequisite: Basic Algebra

Prepared by: Nathaniel M. Cabansay

BSCpE-1 (1st Year BS Computer Engineering)

Cebu Institute of Technology – University


LICENSING

NON-COMMERCIAL

This set of modules is not for sale and is freely distributed. It is also used for educational purposes only.

LICENSING

This work is licensed under the Creative Commons Attribution-NonCommercial-ShareAlike 4.0 International
License. To view a copy of this license, visit http://creativecommons.org/licenses/by-nc-sa/4.0/.
Prepared by: Nathaniel M. Cabansay, BSCpE

TABLE OF CONTENTS

CHAPTER 1 EVALUATING BASIC LIMITS...................................................................... 1

Module 1 Limit Basics ....................................................................................................................................................... 1


Intuitive Definition of a Limit .......................................................................................................................................................... 1
Evaluating Limits Part 1 ..................................................................................................................................................................... 1
Limit Theorems................................................................................................................................................................................... 2
Evaluating Limits Part 2 ..................................................................................................................................................................... 3

Module 2 One-sided Limits ............................................................................................................................................. 6


One-sided Limits ................................................................................................................................................................................ 6
Evaluating One-sided Limits Part 1 ................................................................................................................................................ 6
Existence of the Limit from Both Sides ........................................................................................................................................ 7
Evaluating One-sided Limits Part 2 ................................................................................................................................................ 7

Module 3 Limits to Infinity and Infinite Limits ............................................................................................................ 9


Infinite Limits ....................................................................................................................................................................................... 9
Limits to Infinity .................................................................................................................................................................................. 9
Evaluating Infinite Limits Part 1....................................................................................................................................................... 9
Evaluating Limits at Infinity Part 1 ................................................................................................................................................11
Infinite Limits and Limits to Infinity of Various Functions ..................................................................................................... 14
Evaluating Infinite Limits Part 2..................................................................................................................................................... 17
Evaluating Limits to Infinity Part 2 ............................................................................................................................................... 18

Module 4 Special Limits.................................................................................................................................................. 22


Special Limits ..................................................................................................................................................................................... 22
Proving Special Limits ...................................................................................................................................................................... 22

Module 5 More on Special Limits ................................................................................................................................ 27


Special Limits Involving Hyperbolic Functions ..........................................................................................................................27
Proving the Two Other Special Limits ....................................................................................................................................... 27
Putting it All Together .................................................................................................................................................................... 28
Word Problems Involving Limits .................................................................................................................................................. 32

CHAPTER 2 CONTINUITY ............................................................................................... 36

Module 6 Continuity of a Function ............................................................................................................................. 36


Continuity at a Number ................................................................................................................................................................. 36
Determining Continuity of Functions Part 1............................................................................................................................. 36
Continuity of Particular Functions ............................................................................................................................................... 39
Continuity of Compositions of Functions ................................................................................................................................. 39
Prepared by: Nathaniel M. Cabansay, BSCpE

Determining Continuity of Functions Part 2............................................................................................................................. 40

Module 7 Continuity over an Interval ........................................................................................................................ 43


Continuity over an Open Interval ................................................................................................................................................ 43
Determining Continuity over an Interval Part 1 ...................................................................................................................... 43
Continuity over a Closed Interval ................................................................................................................................................43
Determining Continuity over an Interval Part 2 ...................................................................................................................... 43

Module 8 Types of Discontinuity................................................................................................................................. 45


Discontinuities...................................................................................................................................................................................45
Dealing with Removable Discontinuities .................................................................................................................................... 45
Dealing with Non-Removable Discontinuities .......................................................................................................................... 47

Module 9 More on Discontinuities ............................................................................................................................. 51


Multiple Discontinuities .................................................................................................................................................................. 51
Dealing with Multiple Discontinuities .........................................................................................................................................51
Continuity Problems .......................................................................................................................................................................52
Examples of Continuity Problems ............................................................................................................................................... 53

Module 10 The Intermediate and Extreme Value Theorems ............................................................................... 55


Intermediate Value Theorem ........................................................................................................................................................ 55
Examples of the Intermediate Value Theorem .........................................................................................................................55
Extreme Value Theorem ................................................................................................................................................................55

CHAPTER 3 MORE LIMITS ............................................................................................... 56

Module 11 Limits of Exponential and Logarithmic Functions ............................................................................... 56


Limits of Exponential and Logarithmic Functions .................................................................................................................... 56
Evaluating Limits of Exponential and Logarithmic Functions ................................................................................................ 56

Module 12 Limits of Trigonometric Functions ......................................................................................................... 58


Limits of Trigonometric Functions ..............................................................................................................................................58
Evaluating Limits of Trigonometric Functions .......................................................................................................................... 58

Module 13 Limits of Inverse Trigonometric Functions .......................................................................................... 60


Limits of Inverse Trigonometric Functions................................................................................................................................ 60
Evaluating Limits of Trigonometric Functions .......................................................................................................................... 60

Module 14 Limits of Hyperbolic Functions ............................................................................................................... 62


Limits of Hyperbolic Functions ..................................................................................................................................................... 62
Evaluating Limits of Hyperbolic Functions .................................................................................................................................62

Module 15 Limits of Inverse Hyperbolic Functions................................................................................................. 64


Prepared by: Nathaniel M. Cabansay, BSCpE

Limits of Inverse Hyperbolic Functions ...................................................................................................................................... 64


Evaluating Limits of Inverse Hyperbolic Functions .................................................................................................................. 64

Module 16 Proofs of Limit Properties........................................................................................................................ 65


Epsilon-Delta (ε, δ) Definition of a Limit ................................................................................................................................... 65
Proving a Limit using (ε, δ) Definition.........................................................................................................................................65
Proving the Basic Limit Theorems ............................................................................................................................................... 66
Epsilon-Delta Definitions of Other Kinds of Limits ................................................................................................................74
Proving the Useful Facts of Limits to Infinity ............................................................................................................................ 75

REFERENCES ....................................................................................................................... 78

EXTRAS................................................................................................................................. 79

About This Set of Modules ........................................................................................................................................... 79

Some Quotes to Live By................................................................................................................................................ 79

Non-Commercial ................................................................................................ Error! Bookmark not defined.

Licensing ................................................................................................................ Error! Bookmark not defined.


Prepared by: Nathaniel M. Cabansay, BSCpE

CHAPTER 1 EVALUATING BASIC LIMITS

MODULE 1 LIMIT BASICS

INTUITIVE DEFINITION OF A LI MIT


A limit 𝐿 is a value that a certain function 𝑓(𝑥) approaches as the variable used in the function 𝑥 approaches a
certain value 𝑎. This is written below as:

lim 𝑓(𝑥) = 𝐿

We do not need to have the function defined at 𝑎 but we can have 𝑥 get closer and closer to 𝑎 so we could see
that 𝑓(𝑥) gets closer and closer to 𝐿.

Here’s an example:

lim 𝑥 + 2𝑥 + 3

This means, as 𝑥 gets closer and closer to 2, the function 𝑥 + 2𝑥 + 3 gets closer and closer to some value.

EVALUATING LIMITS PART 1


The limit can be evaluated in three common ways:

1. Using a graph
2. Using a table of values
3. Through limit theorems

Let’s try evaluating lim 𝑥 + 2𝑥 + 3 using a graph.


1
Prepared by: Nathaniel M. Cabansay, BSCpE

Now let’s try evaluating lim 𝑥 + 2𝑥 + 3 using a table of values.


𝒙 𝒇(𝒙) = 𝒙𝟐 + 𝟐𝒙 + 𝟑
1.9 10.41
1.99 10.9401
1.999 10.994001
2.001 11.006001
2.01 11.0601
2.1 11.61
Does the function seem to approach 11 as x approached 2? We’ll find out.

LIMIT THEOREMS

In finding limits, you could use a graph or a table of values, but a graph may take too long and a table of values
might be too much work for you. Fortunately, there are limit theorems you can use to speed up calculations.

Let 𝑓(𝑥), 𝑔(𝑥), and ℎ(𝑥) be functions of 𝑥, 𝑐 be a constant, and 𝐿 = lim 𝑓(𝑥), 𝑀 = lim 𝑔(𝑥), and 𝑁 = lim ℎ(𝑥).
→ → →

1. Limit of a Constant (Theorem 1 from this point on)


lim 𝑐 = 𝑐

2. Identity Limit (Theorem 2 from this point on)


lim 𝑥 = 𝑎

3. Constant Multiple Theorem (Theorem 3 from this point on)


lim 𝑐𝑓(𝑥) = 𝑐 lim 𝑓(𝑥) = 𝑐𝐿
→ →

4. Limit of a Sum or Difference (If combined, Theorem 4 from this point on. If separated, Limit of a Sum is
Theorem 4a and Limit of a Difference is Theorem 4b)
lim [𝑓(𝑥) ± 𝑔(𝑥)] = lim 𝑓(𝑥) ± lim 𝑔(𝑥) = 𝐿 ± 𝑀
→ → →

5. Limit of a Product (Theorem 5 from this point on)


lim [𝑓(𝑥) ⋅ 𝑔(𝑥)] = lim 𝑓(𝑥) ⋅ lim 𝑔(𝑥) = 𝐿𝑀
→ → →

6. Limit of a Quotient (Theorem 6 from this point on)


𝑓(𝑥) lim 𝑓(𝑥) 𝐿
lim = → = ,𝑀 ≠ 0
→ 𝑔(𝑥) lim 𝑔(𝑥) 𝑀

7. Limit of a Power/Radical (Theorem 7 from this point on. Recall that a radical can be written as a rational power)

lim [𝑓(𝑥)] = lim [𝑓(𝑥)] = lim 𝑓(𝑥) = √𝐿 = 𝐿


→ → →

2
Prepared by: Nathaniel M. Cabansay, BSCpE

7a. Limit of a Power (Theorem 7a from this point on)

lim [𝑓(𝑥)] = lim 𝑓(𝑥) =𝐿


→ →

7b. Limit of a Radical/Root (Theorem 7b from this point on)

lim 𝑓(𝑥) = lim 𝑓(𝑥) = √𝐿 = 𝐿


→ →

8. Substitution Theorem (Theorem 8 from this point on)


If 𝑓(𝑥) is continuous at 𝑎, then lim 𝑓(𝑥) = 𝑓(𝑎).

NOTE: Theorem 8 is commonly used with polynomial, sine, and cosine functions.

9. Limit of a Composition of Functions (Theorem 9 from this point on)


If 𝑓(𝑥) is continuous at lim 𝑔(𝑥) = 𝑀, then lim 𝑓(𝑔(𝑥)) = 𝑓 lim 𝑔(𝑥) = lim 𝑓(𝑥) = 𝑓(𝑀).
→ → → →
NOTE: Theorem 9 is commonly used with logarithmic, exponential, trigonometric (other than sine and cosine),
inverse trigonometric, hyperbolic, and inverse hyperbolic functions, as will be seen in Module 3.

10. Squeeze Theorem (Theorem 10 from this point on)


If 𝑓(𝑥) ≤ 𝑔(𝑥) ≤ ℎ(𝑥) and lim 𝑓(𝑥) = lim ℎ(𝑥) or 𝐿 = 𝑁, then lim 𝑔(𝑥) = lim 𝑓(𝑥) = lim ℎ(𝑥) or 𝑀 = 𝐿 = 𝑁.
→ → → → →

Commonly, only the first eight theorems are taught in Calculus courses. I decided to include the last two
theorems here as these can be used to derive other limits, as we will see later in this module, in Module 3, and in
Module 4. The proofs of these theorems will be shown in Module 16. Theorems 8 and 9 will be discussed in more
detail in Module 6.

EVALU ATING LIMITS PART 2

Now, let’s try using these limit theorems to evaluate our example above.

lim 𝑥 + 2𝑥 + 3

No. STATEMENT REASON


lim 𝑥 + 2𝑥 + 3
1 → Given
lim 𝑥 + lim 2𝑥 + lim 3
2 → → → Theorem 4a (Limit of a Sum)
lim 𝑥 + 2 lim 𝑥 + lim 3
3 → → → Theorem 3 (Constant Multiple Rule)

4 (2) + 2(2) + 3 Theorems 1 (Limit of a Constant), 8 (Substitution Theorem)/7a


(Limit of a Power), and 2 (Identity Limit)

5 4+4+3 Square 2 and multiply 2 by 2

6 lim 𝑥 + 2𝑥 + 3 = 11 Add 4, 4, and 3 together


3
Prepared by: Nathaniel M. Cabansay, BSCpE

Then let’s try another limit that’s a bit trickier than the first.

𝑥 − 8𝑥 + 12
lim
→ 𝑥−6

You’d be tempted to use Theorem 6 (Limit of a Quotient), but if you try evaluating:

(6) − 8(6) + 12
=
(6) − 6

36 − 48 + 12
=
0
0
=
0

So, what is ? Is it 0, 1, or ∞? In that case, we call an indeterminate form. There are seven indeterminate forms:

0 ∞
, , 0 ⋅ ∞, ∞ − ∞, 0 , 1 , and ∞ are all indeterminate forms.
0 ∞

If we get any of these, there’s usually another way to get the limit. Now, let’s try that example again, using another
method.

𝑥 − 8𝑥 + 12
lim
→ 𝑥−6

No. STATEMENT REASON


𝑥 − 8𝑥 + 12
1 lim Given
→ 𝑥−6
(𝑥 − 6)(𝑥 − 2)
2 lim
→ 𝑥−6 𝑥 − 8𝑥 + 12 = (𝑥 − 6)(𝑥 − 2)
lim 𝑥 − 2
3 → Divide (𝑥 − 6)(𝑥 − 2) by 𝑥 − 6

4 6−2 Theorems 1 and 2

𝑥 − 8𝑥 + 12
lim = 4
5 → 𝑥−6 Subtract 2 from 6

Now, here’s an even trickier example:

𝑥 − 23
lim
→ √𝑥 + 2 − 5

No. STATEMENT REASON


𝑥 − 23
1 lim Given
→ √𝑥 + 2 − 5

4
Prepared by: Nathaniel M. Cabansay, BSCpE

No. STATEMENT REASON

2 We get an indeterminate limit if we substitute 23 directly and


𝑥 − 23 √𝑥 + 2 + 5
lim ⋅ we cannot factor this one, so we multiply by the conjugate of
→ √𝑥 + 2 − 5 √𝑥 + 2 + 5 the denominator

3 (𝑥 − 23)(√𝑥 + 2 + 5)
lim Multiply to get a single expression
→ (𝑥 + 2) − 25

4 (𝑥 − 23) √𝑥 + 2 + 5
lim Subtract 25 from 2
→ 𝑥 − 23

5 lim √𝑥 + 2 + 5

Divide (𝑥 − 23)(√𝑥 + 2 + 5) by 𝑥 − 23

6 (23) + 2 + 5 Theorems 7b (Limit of a radical), 4a, and 1

7 √25 + 5 Add 2 to 23

8 5+5 Get the principal square root of 25.

9 𝑥 − 23
lim = 10
→ √𝑥 + 2 − 5 Add 5 to 5

Finally, a very tricky example:

𝜋
lim 𝑥 sin
→ 𝑥

No. STATEMENT REASON

1 𝜋 Given
lim 𝑥 sin
→ 𝑥
2 −1 ≤ sin(𝑥) ≤ 1 We cannot do any direct evaluation, factoring or multiplication
by a conjugate. So, we use the bounds of the sine function.

3 𝜋 Changing the argument in the sine function does not change the
−1 ≤ sin≤1
𝑥 bounds of the function.
𝜋
4 −𝑥 ≤ 𝑥 sin ≤𝑥 Multiply all parts of the inequality by 𝑥
𝑥
lim −𝑥 = 0 , lim 𝑥 = 0
5 → → Get the limits of −𝑥 and 𝑥 as 𝑥 approaches 0.

6 𝜋 Theorem 10 (Squeeze Theorem)


lim 𝑥 sin = 0
→ 𝑥

5
Prepared by: Nathaniel M. Cabansay, BSCpE

MODULE 2 ONE-SIDED LIMITS

ONE-SIDED LIMI TS

In the basic form of a limit lim 𝑓(𝑥) = 𝐿, we get a limit from both sides of 𝑎 (as we had seen in the table of values

in the previous module). Sometimes, we want to know what is happening as 𝑥 approaches 𝑎 from only one side.
These are called one-sided limits and come in two forms.

Let 𝐿 represent the limit as 𝑥 approaches 𝑎 from the left, and 𝐿 represent the limit as 𝑥 approaches 𝑎 from the
right. We have:

lim 𝑓(𝑥) = 𝐿 and lim 𝑓(𝑥) = 𝐿


→ →

The left-hand limit is the limit as 𝑥 approaches 𝑎 from the left. It is indicated by a little minus sign (-) above the
𝑎. The right-hand limit is the limit as 𝑥 approaches 𝑎 from the right. It is indicated by a little plus sign (+) above
the 𝑎.

EVALU ATING ONE-SIDED LIMITS PART 1

Example for left-hand limit using a table of values:

lim 𝑥 + 2𝑥 + 3

𝒙 𝒇(𝒙) = 𝒙𝟐 + 𝟐𝒙 + 𝟑
1.9 10.41
1.99 10.9401
1.999 10.994001
1.9999 10.99940001
1.99999 10.9999400001
1.999999 10.999994000001
It seems we’re approaching 11, so: lim 𝑥 + 2𝑥 + 3 = 11

Now let’s find the right-hand limit using a table of values:

lim 𝑥 + 2𝑥 + 3

𝒙 𝒇(𝒙) = 𝒙𝟐 + 𝟐𝒙 + 𝟑
2.000001 11.000006000001
2.00001 11.0000600001
2.0001 11.00060001
2.001 11.006001
2.01 11.0601

6
Prepared by: Nathaniel M. Cabansay, BSCpE

2.1 11.61
It seems the value is also approaching 11, so: lim 𝑥 + 2𝑥 + 3 = 11

EXISTENCE OF THE LIMIT FROM BOTH SIDES

In the previous example, we saw that both the left-hand and right-hand limits generated the same value, which is
11. The previous module showed us that lim 𝑥 + 2𝑥 + 3 = 11. What does that indicate?

If lim 𝑓(𝑥) = lim 𝑓(𝑥) , then lim 𝑓(𝑥) exists and lim 𝑓(𝑥) = lim 𝑓(𝑥) = lim 𝑓(𝑥).
→ → → → → →

Basically, if the left-hand and right-hand limits are equal, then the limit from both sides exists and is equal to both
of the one-sided limits.

If lim 𝑓(𝑥) ≠ lim 𝑓(𝑥) , then lim 𝑓(𝑥) does not exist.
→ → →

Similarly, if the one-sided limits do not equal, then the limit from both sides will not exist. This will commonly
happen with piecewise functions, even roots of odd powers, and logarithmic functions.

EVALU ATING ONE-SIDED LIMITS PART 2


Now, let’s try a piecewise function such as this:

|𝑥|
lim 𝑓(𝑥) , 𝑓(𝑥) = 𝑥 , 𝑥≠0

0 , 𝑥=0

We start by getting the one-sided limits:

lim 𝑓(𝑥) , and lim 𝑓(𝑥)


→ →

|𝒙| |𝒙|
𝒙 𝒇(𝒙) = 𝒙 𝒇(𝒙) =
𝒙 𝒙
-1 -1 1 1
-0.1 -1 0.1 1
-0.01 -1 0.01 1
-0.001 -1 0.001 1
-0.0001 -1 0.0001 1
It seems the function is approaching two different values as 𝑥 approaches 0. When 𝑥 approaches 0 from the left,
𝑓(𝑥) appears to be approaching -1, but when 𝑥 approaches 0 from the right, 𝑓(𝑥) approaches 1 instead. Since the
one-sided limits don’t equal, lim 𝑓(𝑥) does not exist.

Now let’s try another piecewise function:

7 − 4𝑥, 𝑥 < 1
lim 𝑓(𝑥) , 𝑓(𝑥) =
→ 𝑥 + 2, 𝑥 ≥ 1

7
Prepared by: Nathaniel M. Cabansay, BSCpE

Once again, we start by getting the one sided limits.

lim 𝑓(𝑥) = lim 7 − 4𝑥 (This is the limit defined to the left of 1)


→ →

lim 𝑓(𝑥) = lim 𝑥 + 2 (This is the limit defined to the right of 1)


→ →

𝒙 𝒇(𝒙) = 𝟕 − 𝟒𝒙 𝒙 𝒇(𝒙) = 𝒙𝟐 + 𝟐
0.75 4 1.25 3.5625
0.9 3.4 1.1 3.21
0.99 3.04 1.01 3.0201
0.999 3.004 1.001 3.002001
0.9999 3.0004 1.0001 3.00020001
We find that lim 𝑓(𝑥) = 3 and lim 𝑓(𝑥) = 3. Since both one-sided limits are equal, then the limit from both
→ →
sides will exist and lim 𝑓(𝑥) = 3.

Now, let’s try a square root:

lim √𝑥

Notice that we can’t define square roots of negative numbers (in the set of real numbers anyway; complex
numbers will be dealt with in a separate set of modules), so we can only take one of the one-sided limits, in this
case, the right-hand limit.

𝒙 𝒇(𝒙) = √𝒙 𝒙 𝒇(𝒙) = √𝒙
Real: (None)
-1 1 1
Complex: 𝑖

Real: (None)
-0.01 0.01 0.1
Complex: 0.1𝑖

Real: (None)
-0.0001 0.0001 0.01
Complex: 0.01𝑖

Real: (None)
-0.000001 0.000001 0.001
Complex: 0.001𝑖

Real: (None)
-0.00000001 0.00000001 0.0001
Complex: 0.0001𝑖

In this case, the limit from both sides will not exist.

Now for a logarithmic function: lim ln(𝑥). This is the same case as above, where we can only define a right-hand

limit, so the limit from both sides won’t exist with this one either.

8
Prepared by: Nathaniel M. Cabansay, BSCpE

MODULE 3 LIMITS TO INFINITY AND INFINITE LIMITS

INFINITE LIMI TS

Infinite limits are limits whose value is either positive or negative infinity.

lim 𝑓(𝑥) = +∞ or lim 𝑓(𝑥) = −∞


→ →

This will also apply to one-sided limits: lim 𝑓(𝑥) = +∞, lim 𝑓(𝑥) = −∞, lim 𝑓(𝑥) = +∞, or lim 𝑓(𝑥) = −∞.
→ → → →

LIMITS TO INFINITY

Limits to infinity are limits as 𝑥 approaches either +∞ or −∞.

lim 𝑓(𝑥) = 𝐿 and lim 𝑓(𝑥) = 𝐿


→ →

These limits are automatically one-sided limits, as there are no values greater than +∞ (meaning lim 𝑓(𝑥) is a

left-hand limit), and there are no values less than −∞ (meaning lim 𝑓(𝑥) is a right-hand limit). Limits to infinity

can also be infinite limits if lim 𝑓(𝑥) = ±∞ (From this module on, ±∞ will be used here to mean +∞ or −∞).
→±

In evaluating limits to infinity, here are a few useful facts before we go on evaluating them:

1
lim =0
→± 𝑥

Basically, as 𝑥 gets larger and larger or smaller and smaller, gets closer and closer to 0.

If 𝑝(𝑥) is a polynomial function 𝑎 𝑥 + 𝑎 𝑥 + ⋯ + 𝑎 𝑥 + 𝑎 , then lim 𝑝(𝑥) = lim 𝑎 𝑥 .


→± →±

In evaluating a limit to infinity of a polynomial function, get the limit of the term with the highest exponent. The
proofs of these two facts above will be shown in Module 16.

EVALU ATING INFINITE LIMITS PART 1

Example for an infinite limit:

1
lim
→ 𝑥

We start by getting the one-sided limits, as we must see what happens as 𝑥 approaches 0 from both sides.

𝟏 𝟏
𝒙 𝒇(𝒙) = 𝒙 𝒇(𝒙) =
𝒙 𝒙
-1 -1 1 1
-0.1 -10 0.1 10
-0.01 -100 0.01 100

9
Prepared by: Nathaniel M. Cabansay, BSCpE

-0.001 -1000 0.001 1000


-0.0001 -10000 0.0001 10000
From the left of 0, the value appears to be getting smaller and smaller without bound as 𝑥 approaches 0. From the
right of 0, however, the value appears to be getting larger and larger without bound instead. The one-sided limits
are: lim = −∞ and lim = +∞. However, the one-sided limits don’t equal, so lim does not exist.
→ → →

Now let’s try a different limit:

1
lim
→ 𝑥

𝟏 𝟏
𝒙 𝒇(𝒙) = 𝒙 𝒇(𝒙) =
𝒙𝟐 𝒙𝟐
-1 1 1 1
-0.1 100 0.1 100
-0.01 10 000 0.01 10 000
-0.001 1 000 000 0.001 1 000 000
-0.0001 100 000 000 0.0001 100 000 000
This time, the value appears to be getting larger and larger as 𝑥 approaches 0 from both the left and the right. We
can then conclude that the one-sided limits are lim = +∞ and lim = +∞. Since the two one-sided limits
→ →
equal, we now conclude lim = +∞.

Another example:

𝑥
lim
→ 𝑥−1

Again, we get the one-sided limits first.

𝒙 𝒙
𝒙 𝒇(𝒙) = 𝒙 𝒇(𝒙) =
𝒙−𝟏 𝒙−𝟏
0 -1 2 1
0.9 -9 1.1 11
0.99 -99 1.01 101
0.999 -999 1.001 1 001
0.9999 -9 999 1.0001 10 001
The one-sided limits are: lim = −∞ and lim = +∞. As the two limits don’t equal, the limit from both
→ →
sides does not exist.

Now, let’s try a one-sided limit (Note: lg 𝑥 = log 𝑥 = log 𝑥):

lim lg 𝑥

10
Prepared by: Nathaniel M. Cabansay, BSCpE

𝒙 𝒇(𝒙) = 𝐥𝐠(𝒙)
1 0
10-10 -10
10-100 -100
10-1 000 -1 000
10-10 000 -10 000
The values seem to be decreasing and decreasing without bound as 𝑥 approaches 0 from the right. We can then
conclude that lim lg(𝑥) = −∞.

Finally, let’s try a limit of a trigonometric function:

lim tan(𝑥) and lim tan(𝑥)


→ →

This time, we’ll use the graph of the function.

You’ll notice that from the left of , the graph of the function appears to be heading to +∞, while from the right
of , the graph appears to be heading to −∞ instead. This means lim tan(𝑥) = +∞, lim tan(𝑥) = −∞ and
→ →

lim tan(𝑥) does not exist.


EVALUATING LIMITS AT INFINITY PART 1

For example:

lim 𝑥 + 2𝑥 + 3

11
Prepared by: Nathaniel M. Cabansay, BSCpE

All we need to do is get lim 𝑥 , which is +∞, so lim 𝑥 + 2𝑥 + 3 = +∞.


→ →

Now for another example:

lim 𝑥 − 𝑥 + 6

All we need to do is get lim 𝑥 , which is −∞, so lim 𝑥 − 𝑥 + 6 = −∞.


→ →

A more complicated example:

8 − 4𝑥
lim
→ 9𝑥 + 5𝑥

For rational functions, we look at the highest exponent of the denominator (not the numerator). So:

No. STATEMENT REASON


8 − 4𝑥
1 lim
→ 9𝑥 + 5𝑥 Given

8 We get an indeterminate limit if we substitute +∞ directly, so


𝑥 −4
2 lim 𝑥 we factor out the highest power of 𝑥 of the denominator, in
→ 5
𝑥 9+
𝑥 this case 𝑥 .
8
−4
3 lim 𝑥 Simplify the expression
→ 5
9+
𝑥
8
lim −4
→ 𝑥
4 5 Theorem 6 (Limit of a Quotient)
lim 9 +
→ 𝑥
8
lim − lim 4
𝑥
5 → →
Theorem 4 (Limit of a Sum or Difference)
5
lim 9 + lim
→ → 𝑥
0−4
6 lim = 0, and Theorems 1 and 3.
9+0 →±

8 − 4𝑥 4
lim = −
7 → 9𝑥 + 5𝑥 9 Add 0 to 9 and subtract 4 from 0.

And another more complicated example:

3𝑥 − 4𝑥 + 𝑥
lim
→ 1 − 2𝑥

No. STATEMENT REASON


3𝑥 − 4𝑥 − 𝑥
1 lim Given
→ 1 − 2𝑥

12
Prepared by: Nathaniel M. Cabansay, BSCpE

No. STATEMENT REASON

𝑥 (3𝑥 − 4𝑥 − 1) We get an indeterminate limit if we substitute −∞ directly, so


2 lim we factor out the highest power of 𝑥 of the denominator, in
→ 1
𝑥 −2 this case 𝑥 .
𝑥
3𝑥 − 4𝑥 − 1
lim
3 → 1
−2 Simplify the expression
𝑥
lim (3𝑥 − 4𝑥 − 1)

4 1 Theorem 6
lim −2
→ 𝑥
+∞ lim (𝑎 𝑥 + 𝑎 𝑥 + ⋯ + 𝑎 𝑥 + 𝑎 ) = lim 𝑎 𝑥 and
→ →
5 1 Theorem 4b (Limit of a Difference)
lim − lim 2
→ 𝑥 →
+∞
6 lim = 0, and Theorems 1 and 3.
2 →±

3𝑥 − 4𝑥 − 𝑥
lim = +∞
7 → 1 − 2𝑥 = +∞, 𝑛 ≠ 0 or 𝑛 ≠ ±∞

And one final complicated example:

𝑥 − 3𝑥 + 6
lim
→ 2𝑥 − 8𝑥

No. STATEMENT REASON


𝑥 − 3𝑥 + 6
1 lim
→ 2𝑥 − 8𝑥 Given

1 3 6
𝑥 − + Factor out the highest power of 𝑥 of the denominator, in this
2 lim 𝑥 𝑥 𝑥
→ 2 case 𝑥 .
𝑥 −8
𝑥
1 3 6
− +
lim 𝑥 𝑥 𝑥
3 → 2 Simplify the expression
−8
𝑥
1 3 6
lim − +
→ 𝑥 𝑥 𝑥
4 2 Theorem 6
lim −8
→ 𝑥
1 3 6
lim − lim − lim
5 → 𝑥 → 𝑥 → 𝑥 Theorem 4b
1
lim − lim 2
→ 𝑥 →
𝑥 − 3𝑥 + 6
lim = 0
6 → 2𝑥 − 8𝑥 lim = 0, and Theorems 1 and 3

13
Prepared by: Nathaniel M. Cabansay, BSCpE

INFINITE LIMI TS AND LIMITS TO INFINITY OF VARIOUS FUNCTI ONS

Here are infinite limits and limits to infinity of common classes of functions.

1. Powers of x (Let 𝑛, 𝑞, 𝑟 ∈ ℤ (meaning 𝑛, 𝑞 and 𝑟 are all integers) where 𝑞 is an odd integer and 𝑟 is an
even integer)
a. Integer Powers of x
1 +∞ , 𝑛 > 0
lim 𝑥 = lim =
→ → 𝑥 0 , 𝑛<0

1 +∞ , 𝑛 > 0 and is even


lim 𝑥 = lim = −∞ , 𝑛 > 0 and is odd
→ → 𝑥 0 , 𝑛<0

1 0 , 𝑛>0
lim 𝑥 = lim =
→ → 𝑥 +∞ , 𝑛<0

1 0 , 𝑛>0
lim 𝑥 = lim = +∞ , 𝑛 < 0 and is even
→ → 𝑥 −∞ , 𝑛 < 0 and is odd
b. Odd Roots of 𝑥

𝑛
+∞ , >0
1 𝑞
lim √𝑥 = lim = 𝑛
→ → √𝑥 0 , <0
𝑞
𝑛
⎧+∞ , 𝑞 > 0 and 𝑛 is even
1 ⎪ 𝑛
lim √𝑥 = lim = −∞ , > 0 and 𝑛 is odd
→ → √𝑥 ⎨ 𝑞
⎪ 0 , 𝑛<0
⎩ 𝑞

𝑛
0 , >0
1 𝑞
lim √𝑥 = lim = 𝑛
→ → √𝑥 +∞ , <0
𝑞
𝑛
⎧ 0 , >0
𝑞
1 ⎪ 𝑛
lim √𝑥 = lim = +∞ , < 0 and 𝑛 is even
→ → √𝑥 ⎨ 𝑞
⎪−∞ 𝑛
, < 0 and 𝑛 is odd
⎩ 𝑞

c. Even Roots of 𝑥
𝑛
1 +∞ , >0
lim √𝑥 = lim = 𝑟
𝑛
→ → √𝑥 0 , <0
𝑟

14
Prepared by: Nathaniel M. Cabansay, BSCpE

𝑛
⎧ +∞ , > 0 and 𝑛 is even
1 ⎪ 𝑟
lim √𝑥 = lim = 𝑛
→ → √𝑥 ⎨ 0 , < 0 and 𝑛 is even
⎪ 𝑟
⎩does not exist , 𝑛 is odd

𝑛
1 >0 0 ,
lim √𝑥 = lim = 𝑟
𝑛
→ → √𝑥 +∞ , <0
𝑟
𝑛
⎧ 0 , > 0 and 𝑛 is even
1 ⎪ 𝑟
lim √𝑥 = lim = 𝑛
→ → √𝑥 ⎨ +∞ , < 0 and 𝑛 is even
⎪ 𝑟
⎩does not exist , 𝑛 is odd
2. Exponential Functions (Let 𝑎 ∈ ℝ (meaning 𝑎 is a real number))
1 +∞ , 𝑎 > 1
lim 𝑎 = lim =
→ → 𝑎 0 , 1>𝑎>0
1 0 , 𝑎>1
lim 𝑎 = lim =
→ → 𝑎 +∞ , 1 > 𝑎 > 0
3. Logarithmic Functions
1 +∞ , 𝑎 > 1
lim log (𝑥) = lim log =
→ → 𝑥 −∞ , 1 > 𝑎 > 0
1 −∞ , 𝑎 > 1
lim log (𝑥) = lim log =
→ → 𝑥 +∞ , 1 > 𝑎 > 0

More limits involving exponential functions, and logarithmic functions will be detailed in Module 11.

4. Trigonometric Functions (Let 𝑘 ∈ ℤ)


lim tan(𝑥) = lim cot(𝑥) = +∞
→( )

lim tan(𝑥) = lim cot(𝑥) = −∞


→( )

lim csc(𝑥) = lim csc(𝑥) = +∞


→([ ] ) →( )
lim csc(𝑥) = lim csc(𝑥) = −∞
→([ ] ) →( )
lim sec(𝑥) = lim sec(𝑥) = +∞
→ →

lim sec(𝑥) = lim sec(𝑥) = −∞


→ →

More limits involving trigonometric functions will be detailed in Module 12.

5. Inverse Trigonometric Functions


𝜋
lim arctan(𝑥) = lim arccot(𝑥) =
→ → 2
𝜋
lim arctan(𝑥) = lim arccot(𝑥) = −
→ → 2
lim arccot(𝑥) = lim arctan(𝑥) = 0
→± →
lim arccsc(𝑥) = lim arcsin(𝑥) = 0
→± →
𝜋
lim arcsec(𝑥) = lim arccos(𝑥) =
→± → 2

15
Prepared by: Nathaniel M. Cabansay, BSCpE

More limits involving inverse trigonometric functions will be detailed in Module 13.

Before we continue on, let’s define hyperbolic functions a bit. A hyperbolic function is defined similarly to a
trigonometric function, but instead of the function being defined on the unit circle 𝑥 + 𝑦 = 1, the function is
defined on the unit hyperbola 𝑥 − 𝑦 = 1. The two most basic hyperbolic functions are:

e −e e +e
sinh 𝑡 = and cosh 𝑡 =
2 2

Four other hyperbolic functions can be defined with the two basic hyperbolic functions above:

sinh 𝑡 e − e
tanh 𝑡 = =
cosh 𝑡 e + e

1 2
csch 𝑡 = =
sinh 𝑡 e − e
1 2
sech 𝑡 = =
cosh 𝑡 e + e

cosh 𝑡 e + e
coth 𝑡 = =
sinh 𝑡 e − e

Just as trigonometric functions have inverses, hyperbolic functions also have inverses (which measure the area
between the hyperbola and the line from the origin to a point on the hyperbola):

arsinh 𝑡 = ln 𝑡 + 𝑡 + 1

arcosh 𝑡 = ln 𝑡 + 𝑡 − 1

1 1+𝑡
artanh 𝑡 = ln
2 1−𝑡

1 1
arcsch 𝑡 = ln + +1
𝑡 𝑡

1 1
arsech 𝑡 = ln + −1
𝑡 𝑡

1 𝑡+1
arcoth 𝑡 = ln
2 𝑡−1

The proof of these definitions will be covered in a different set of modules dedicated to them.

6. Hyperbolic Functions

lim sinh(𝑥) = lim csch(𝑥) = +∞


→ →
lim sinh(𝑥) = lim csch(𝑥) = −∞
→ →

16
Prepared by: Nathaniel M. Cabansay, BSCpE

lim csch(𝑥) = lim sinh(𝑥) = 0


→± →
lim cosh(𝑥) = +∞
→±
lim sech(𝑥) = 0
→±
lim tanh(𝑥) = lim coth(𝑥) = 1
→ →
lim tanh(𝑥) = lim coth(𝑥) = −1
→ →
lim coth(𝑥) = +∞

lim coth(𝑥) = −∞

More limits involving hyperbolic functions will be detailed in Module 14.

7. Inverse Hyperbolic Functions

lim arsinh(𝑥) = lim arcsch(𝑥) = +∞


→ →
lim arsinh(𝑥) = lim arcsch(𝑥) = −∞
→ →
lim arcosh(𝑥) = lim arsech(𝑥) = +∞
→ →
lim artanh(𝑥) = lim arcoth(𝑥) = +∞
→ →
lim artanh(𝑥) = lim arcoth(𝑥) = −∞
→ →
lim arcoth(𝑥) = lim artanh(𝑥) = 0
→± →

More limits involving inverse hyperbolic functions will be detailed in Module 15.

To make evaluating limits of these types easier, extend Theorem 9 (Limit of a Composition of Functions) to
include these cases:

1
lim = 0 and lim (𝑎 𝑥 + 𝑎 𝑥 + ⋯ + 𝑎 𝑥 + 𝑎 ) = lim 𝑎 𝑥
→± 𝑥 →± →±

EVALU ATING INFINITE LIMITS PART 2

Let’s try a hyperbolic function this time:

lim 2 coth(𝑥 )

No. STATEMENT REASON


lim 2 coth(𝑥 )
1 → Given
2 lim coth(𝑥 )
2 → Theorem 3

2 coth( lim 𝑥 ) Theorem 9 (Using lim 𝑓 𝑔(𝑥) = 𝑓 lim 𝑔(𝑥) )


3 →
→ →

lim 𝑥 = 0
4 → Theorem 8

5
2 lim coth(𝑥) Theorem 9 (This time using lim 𝑓 𝑔(𝑥) = lim 𝑓(𝑥))
→ → →

2(+∞) lim coth(𝑥) = +∞


6 →

17
Prepared by: Nathaniel M. Cabansay, BSCpE

No. STATEMENT REASON

lim 2 coth(𝑥 ) = +∞
7 →
Multiply 2 by +∞

Now an inverse hyperbolic function:

lim artanh(2𝑥)

No. STATEMENT REASON


lim artanh(2𝑥)
1 → Given
artanh( lim 2𝑥)
2 → Theorem 9
artanh(2 lim 𝑥)
3 → Theorem 3
1
2 lim 𝑥 = 2 =1
4 → 2 Theorem 2

5
lim artanh(𝑥)

Theorem 9

lim artanh(2𝑥) = +∞
6 → lim artanh(𝑥) = +∞

EVALU ATING LIMITS TO INFINI TY PART 2

Let’s try a limit to infinity of an odd root:

lim √𝑥

No. STATEMENT REASON

1 lim √𝑥 Given

2 lim 𝑥 Theorem 7b

3 √−∞ Theorem 2

lim √𝑥 = −∞
4 → Get the cube root of −∞

Another limit to infinity of an odd root:

lim 𝑥

18
Prepared by: Nathaniel M. Cabansay, BSCpE

No. STATEMENT REASON

1 lim 𝑥 Given

2 lim 𝑥 Theorem 7b

3 √+∞ Theorem 8/7a

lim 𝑥 = +∞
4 → Get the fifth root of +∞

Then a limit to infinity of an even root:

lim 𝑥

There are actually two ways you can solve this limit:

First, the traditional way.

No. STATEMENT REASON

1 lim 𝑥 Given

2 lim 𝑥 Theorem 7b

3 √+∞ Theorem 8/7a

lim 𝑥 = +∞
4 → Get the square root of +∞

Second, we’ll use the fact that √𝑥 = |𝑥|

No. STATEMENT REASON

1 lim 𝑥 Given

lim |𝑥|
2 → √𝑥 = |𝑥|

3 lim 𝑥 Theorem 9

4 |−∞| Theorem 2

lim 𝑥 = +∞
5 → Get the absolute value of −∞

And another limit to infinity of an even root:

lim 𝑥

19
Prepared by: Nathaniel M. Cabansay, BSCpE

No. STATEMENT REASON

1 lim 𝑥 Given

2 lim 𝑥 Theorem 7b

3 √−∞ Theorem 8/7a

lim 𝑥 does not exist


4 → Even roots of negative numbers are undefined

Now let’s try an exponential function:

lim e

No. STATEMENT REASON


lim e
1 → Given

2 e → Theorem 9

3 e → Theorem 3
−2 lim 𝑥 = −2(+∞) = −∞
4 → Theorem 2

5
lim e

Theorem 9

lim e = 0 0 , 𝑎>1
6 → lim 𝑎 =
+∞ , 1>𝑎>0
, e > 1, ∴ lim 𝑎 = 0
→ →

This time, let’s try a limit to infinity of an inverse trigonometric function.

lim 4 arctan(3𝑥)

No. STATEMENT REASON


lim 4 arctan(3𝑥)
1 → Given
4 lim arctan(3𝑥)
2 → Theorem 3

3 4 arctan lim 3𝑥 Theorem 9


4 4 arctan 3 lim 𝑥 Theorem 3


3 lim 𝑥 = 3(+∞) = +∞
5 → Theorem 2

6
4 lim arctan(𝑥)

Theorem 9 again
𝜋
7 4 lim arctan(𝑥) =
2 →

20
Prepared by: Nathaniel M. Cabansay, BSCpE

No. STATEMENT REASON

lim 4 arctan(3𝑥) = 2𝜋
8 → Multiply 4 by

Then let’s try a limit to infinity of a hyperbolic function.

lim tanh(4𝑥)

No. STATEMENT REASON


lim tanh(4𝑥)
1 → Given
tanh( lim 4𝑥)
2 → Theorem 9
tanh(4 lim 𝑥)
3 → Theorem 3
4 lim 𝑥 = 4(−∞) = −∞
4 → Theorem 2

5
lim tanh(𝑥)

Theorem 9 again

lim tanh(4𝑥) = −1
6 → lim tanh(𝑥) = −1


Finally, let’s try a limit to infinity of an inverse hyperbolic function.

lim − arsinh(−2𝑥)

No. STATEMENT REASON


lim − arsinh(−2𝑥)
1 → Given
− lim arsinh(−2𝑥)
2 → Theorem 3

3 − arsinh lim −2𝑥 Theorem 9


4 − arsinh −2 lim 𝑥 Theorem 3


−2 lim 𝑥 = −2(−∞) = +∞
5 → Theorem 2

6
− lim arsinh(𝑥)

Theorem 9 again

−(+∞) lim arsinh(𝑥) = +∞


7 →

lim − arsinh(−2𝑥) = −∞
8 → Multiply +∞ by -1.

21
Prepared by: Nathaniel M. Cabansay, BSCpE

MODULE 4 SPECIAL LIMITS

SPECI AL LIMITS

Let us now apply what we learned in Modules 1 to 3 to derive some special limits. These limits, in turn, can be
used to derive other limits and derivatives (in a separate set of modules).

These limits are:

( )
1. lim

( )
2. lim

3. lim 1+
→±

4. lim(1 + 𝑟𝑥)

5. lim

6. lim

PROVING SPECIAL LIMITS

First, let’s derive:

sin(𝑥)
lim
→ 𝑥

Let’s start with a graph

No. STATEMENT REASON

Given (all on Quadrant I):


𝑥 + 𝑦 = 1, O = (0, 0), C = (1, 0), and OC = 1
A line is drawn tangent to the unit circle at point C. A ray is drawn
from point O at an angle θ. We define the point where the ray
intersects the circle as point A. Because point A is on the unit circle,
it is defined with the coordinates
A = (cos 𝜃 , sin 𝜃)
1
We then define the intersection of ray OA and the line tangent to
the circle at point C as point B. Next, we define a point D such that
triangle ODA is a right triangle. Since tangents are always at a right
angle to the radius, triangle OCB is also a right triangle. Using the
coordinates of point A and D we get:
D = (cos 𝜃 , 0)
OD = cos 𝜃
AD = sin 𝜃
AD BC
2 = Use similar triangles to find length BC.
OD OC

22
Prepared by: Nathaniel M. Cabansay, BSCpE

No. STATEMENT REASON


sin 𝜃
3 = BC AD = sin 𝜃 , OD = cos 𝜃 , and OC = 1
cos 𝜃

𝐵𝐶 = tan 𝜃 sin 𝜃
4 = tan 𝜃
cos 𝜃

5 ΔOAC ≤ Sector AOC ≤ ΔOCB Draw segment AC to define triangle OAC. From graph.
1
ΔOCB = (BC)(OC)
2
1 Area of triangle OCB. The base of the triangle is segment OC = 1
6 ΔOCB = (tan 𝜃)(1)
2 and the height of the triangle is segment BC = tan θ.
tan 𝜃
ΔOCB =
2
1
ΔOAC = (AD)(OC) Area of triangle OAC. The base of the triangle is segment OC = 1
2
7 sin 𝜃 and the height of the triangle is segment AD = sin θ.
ΔOAC =
2
1
Sector AOC = 𝑟 𝜃
2 Area of sector AOC, where r = 1.
8 𝜃
Sector AOC =
2
sin 𝜃 𝜃 tan 𝜃
9 ≤ ≤ From statements 5 to 8
2 2 2

10 sin 𝜃 ≤ 𝜃 ≤ tan 𝜃 Multiply all parts of the inequality by 2. True for all angles on QI.

Now we get to the actual proof using the Squeeze Theorem:

No. STATEMENT REASON


sin(𝑥)
1 lim
→ 𝑥 Given

2 sin(𝑥) ≤ 𝑥 ≤ tan(𝑥) Derived from the graph above, with θ = x.


𝑥 1
3 1≤ ≤ Divide all parts of the inequality by sin(𝑥)
sin(𝑥) cos(𝑥)
sin(𝑥)
4 1≥
𝑥
≥ cos(𝑥) Get the reciprocals of all parts of the inequality
sin(𝑥)
5 lim 1 ≥ lim ≥ lim cos(𝑥) On all parts of the inequality, get the limits as x approaches 0.
→ → 𝑥 →

6
lim 1 = 1 𝑎𝑛𝑑 lim cos(𝑥) = 1
→ →
Theorems 1 and 8

𝐬𝐢𝐧(𝒙)
𝐥𝐢𝐦 = 𝟏
7 𝒙→𝟎 𝒙 Theorem 10

Next, let’s derive:

1 − cos(𝑥)
lim
→ 𝑥

23
Prepared by: Nathaniel M. Cabansay, BSCpE

No. STATEMENT REASON


1 − cos(𝑥)
1 lim
→ 𝑥 Given
1 − cos (𝑥)
2 lim Multiply the numerator and denominator by 1 + cos(𝑥)
→ 𝑥(1 + cos(𝑥))

sin 𝑥
3 lim 1 − cos (𝑥) = sin (𝑥)
→ 𝑥(1 + cos(𝑥))
sin(𝑥) sin(𝑥) ( ) ( )
4 lim ⋅ = ⋅
→ 𝑥 1 + cos(𝑥) ( ( )) ( )

sin(𝑥) sin(𝑥)
5 lim ⋅ lim Theorem 5 (Limit of a Product)
→ 𝑥 → 1 + cos(𝑥)

sin(𝑥) sin(𝑥)
6 lim lim =1
→ 1 + cos(𝑥) → 𝑥
lim sin(𝑥)

7 Theorem 6
lim[1 + cos(𝑥)]

lim sin(𝑥)

8 Theorem 4a
lim 1 + lim cos(𝑥)
→ →
0
9 Theorems 1 and 8
2
𝟏 − 𝐜𝐨𝐬(𝒙)
𝐥𝐢𝐦 = 𝟎
10 𝒙→𝟎 𝒙 0 divided by any number except 0 is 0.

Now let’s derive:

𝑟
lim 1+
→± 𝑛

No. STATEMENT REASON


𝑟
1 lim 1 + Given
→± 𝑛
𝑟 (𝑛𝑡)(𝑛𝑡 − 1) 𝑟 (𝑛𝑡)(𝑛𝑡 − 1)(𝑛𝑡 − 2) 𝑟
1 + (𝑛𝑡)
2 lim
→± 𝑛
+
2! 𝑛
+
3! 𝑛
+⋯
Binomial Expansion
1 1 2
(𝑛𝑡) 1 − (𝑛𝑡)
𝑛𝑡 𝑟 1−
𝑛𝑡
1−
𝑛𝑡 𝑟 Factor an 𝑛𝑡 from each binomial term in the
3 lim
→±
1 + 𝑟𝑡 +
2! 𝑛
+
3! 𝑛
+⋯
numerator and multiply to the monomial 𝑛𝑡.
(𝑟𝑡) 1 (𝑟𝑡) 1 2 Simplify the expression by cancelling ns and
lim 1 + 𝑟𝑡 + 1− + 1− 1− +⋯
4 2! 𝑛𝑡 3! 𝑛𝑡 𝑛𝑡
rewriting the expression.
→±

(𝑟𝑡) 1
lim 1 + lim 𝑟𝑡 + lim 1− +⋯
5 →± →± →± 2! 𝑛𝑡 Theorem 4a

(𝑟𝑡) 1 (𝑟𝑡) 1 2 Treat all instances of r and t as constants,


6 1 + 𝑟𝑡 + lim
2! →±
1−
𝑛
+ lim
3! →±
1−
𝑛
lim 1−
𝑛
+⋯
→±
then apply theorems 1, 3 and 5

24
Prepared by: Nathaniel M. Cabansay, BSCpE

No. STATEMENT REASON


(𝑟𝑡) 1 (𝑟𝑡) 1 1
7 1 + 𝑟𝑡 +
2!
1 − lim
→± 𝑛
+
3!
1 − lim
→± 𝑛
1 − 2 lim
→± 𝑛
+⋯ Theorems 1, 3 and 4b
(𝑟𝑡) (𝑟𝑡)
8 1 + 𝑟𝑡 + + +⋯ lim = 0, then multiply all 1s.
2! 3! →±

𝑥 𝑥
9 e =1+𝑥+ + +⋯
2! 3! Series expansion of e
(𝑟𝑡) (𝑟𝑡)
10 e = 1 + 𝑟𝑡 +
2!
+
3!
+ ⋯. Substitute 𝑥 with 𝑟𝑡

𝒓 𝒏𝒕
𝐥𝐢𝐦 𝟏 + = 𝐞𝒓𝒕
11 𝒏→± 𝒏 From statements 8 and 10.

Next, let’s derive:

lim(1 + 𝑟𝑥)

No. STATEMENT REASON

1 lim(1 + 𝑟𝑥) Given



𝑟
2 lim(1 + 𝑟𝑥) = lim 1+ Let 𝑥 = , 𝑛 = . As 𝑥 approaches 0, 𝑛 approaches ±∞
→ →± 𝑛
𝒕
𝐥𝐢𝐦(𝟏 + 𝒓𝒙)𝒙 = 𝐞𝒓𝒕 𝑟
3 𝒙→𝟎 lim 1+ =e
→± 𝑛

Then let’s derive:

𝑎 −1
lim
→ 𝑥

No. STATEMENT REASON


𝑎 −1
1 lim
→ 𝑥 Given

2 𝑦 =𝑎 −1 Let 𝑦 = 𝑎 − 1

3 𝑎 = 1+𝑦 Add 1 to both sides

4 ln 𝑎 = ln(1 + 𝑦) Get the natural logarithm of both sides

5 𝑥 ln 𝑎 = ln(1 + 𝑦) log 𝑥 = 𝑦 log 𝑥


ln(1 + 𝑦)
6 𝑥= Divide both sides by ln 𝑎
ln 𝑎

25
Prepared by: Nathaniel M. Cabansay, BSCpE

No. STATEMENT REASON


lim 𝑦 = lim 𝑎 − 1
→ →
lim 𝑦 = 𝑎 − 1 From statement 2, get the limits of both sides as x approaches 0
7 →
lim 𝑦 = 1 − 1 = 0

𝑦
lim
8 → ln(1 + 𝑦) From statements 6 and 7, rewrite the limit in terms of y.
ln 𝑎
( )
ln 𝑎 Move ln 𝑎 into the numerator by multiplying 𝑦 by ( )
, then
lim
9 → 1
⋅ ln(1 + 𝑦) move 𝑦 into the denominator by rewriting as
𝑦

ln 𝑎
lim
10 → log 𝑥 = 𝑦 log 𝑥
ln(1 + 𝑦)
lim ln 𝑎

11 Theorem 6
lim ln(1 + 𝑦)

ln 𝑎
12 ln lim (1 + 𝑦) Theorems 1 and 9

ln 𝑎
13 ln e lim(1 + 𝑦) = e

𝒂𝒙 − 𝟏
𝐥𝐢𝐦 = 𝐥𝐧 𝒂
14 𝒙→𝟎 𝒙 ln e = 1

Finally, let’s derive:

e −1
lim
→ 𝑥

No. STATEMENT REASON


e −1
1 lim
→ 𝑥 Given

e −1 𝑎 −1
2 lim
→ 𝑥
= ln e lim = ln 𝑎 where 𝑎 = e
→ 𝑥
𝐞𝒙 − 𝟏
𝐥𝐢𝐦 = 𝟏
3 𝒙→𝟎 𝒙 ln e = 1

26
Prepared by: Nathaniel M. Cabansay, BSCpE

MODULE 5 MORE ON SPECIAL LIMITS

SPECIAL LIMITS INVOLVING HYPERBOLIC FUNCTIONS

We will see two more special limits involving hyperbolic functions.

( )
1. lim

( )
2. lim

PROVING THE TWO OTHER SPECIAL LIMI TS

Let’s derive:

sinh(𝑥)
lim
→ 𝑥

No. STATEMENT REASON


sinh(𝑥)
1 lim
→ 𝑥 Given
sinh(𝑥) e −e
2 lim
→ 𝑥
= lim
→ 2𝑥 Exponential Form of Hyperbolic Sine multiplied by
sinh(𝑥) 1 e −e
3 lim
→ 𝑥
= lim
2 → 𝑥 Theorem 3
sinh(𝑥) 1 e −1+1−e
4 lim
→ 𝑥
= lim
2 → 𝑥 Add and subtract 1 from the numerator
sinh(𝑥) 1 e −1 1−e
5 lim = lim + lim Split the fraction and use Theorem 4
→ 𝑥 2 → 𝑥 → 𝑥
sinh(𝑥) 1 e −1 e −1
6 lim = lim + lim Remove the negative exponent by multiplying by
→ 𝑥 2 → 𝑥 → 𝑥e
sinh(𝑥) 1 e −1 1 e −1
7 lim
→ 𝑥
= lim
2 → 𝑥
+ lim
→ e
lim
→ 𝑥 Theorem 5
sinh(𝑥) 1
8 lim
→ 𝑥
= (1 + 1)
2 e = 1 and lim =1

sinh(𝑥) 1
9 lim
→ 𝑥
= (2)
2 Add 1 to 1

𝐬𝐢𝐧𝐡(𝒙)
𝐥𝐢𝐦 = 𝟏
10 𝒙→𝟎 𝒙 Multiply by 2

Now let’s derive:

1 − cosh(𝑥)
lim
→ 𝑥

27
Prepared by: Nathaniel M. Cabansay, BSCpE

No. STATEMENT REASON


1 − cosh(𝑥)
1 lim
𝑥→
Given
1 − cosh(𝑥) 2−e −e
2 lim
→ 𝑥
= lim
→ 2𝑥 Exponential Form of Hyperbolic Sine multiplied by
1 − cosh(𝑥) 1 2−e −e
3 lim
→ 𝑥
= lim
2 → 𝑥 Theorem 3
1 − cosh(𝑥) 1 1−e +1−e
4 lim
→ 𝑥
= lim
2 → 𝑥 Rewrite 2 into 1 + 1
1 − cosh(𝑥) 1 1−e 1−e
5 lim
→ 𝑥
= lim
2 → 𝑥
+ lim
→ 𝑥 Split the fraction and use Theorem 4

1 − cosh(𝑥) 1 e −1 e −1 Remove the negative exponent by multiplying by


6 lim
𝑥
=
2
− lim
𝑥
+ lim
𝑥e
and factor a -1 from and use Theorem 3
→ → →

1 − cosh(𝑥) 1 e −1 1 e −1
7 lim
→ 𝑥
=
2
− lim
→ 𝑥
+ lim
→ e
lim
→ 𝑥 Theorem 5
1 − cosh(𝑥) 1
8 lim
→ 𝑥
= (−1 + 1)
2 e = 1 and lim =1

1 − cosh(𝑥) 1
9 lim
→ 𝑥
= (0)
2 Add 1 to -1

𝟏 − 𝐜𝐨𝐬𝐡(𝒙)
𝐥𝐢𝐦 = 𝟎
10 𝒙→𝟎 𝒙 Multiply by 0

PUTTING IT ALL TOGETHER

Now let’s apply what we learned in Modules 1 to 4

First, let’s try combining a limit we learned in Module 4 with a limit theorem from Module 1:

sin 3𝑥
lim
→ 𝑥

No. STATEMENT REASON


sin(3𝑥)
1 lim
→ 𝑥 Given
3 sin(3𝑥)
2 lim Multiply the numerator and the denominator by 3
→ 3𝑥
sin(3𝑥)
3 3 lim
→ 3𝑥 Theorem 3

3(1) sin(ℎ)
4 lim =1
→ ℎ
sin(3𝑥)
5 lim = 3 Multiply 3 by 1
→ 𝑥

28
Prepared by: Nathaniel M. Cabansay, BSCpE

No. STATEMENT REASON

Now, we’ll see a more complicated limit:

1 − cos (𝑥) + sin (𝑥)


lim
→ 𝑥

No. STATEMENT REASON


1 − cos (𝑥) + sin (𝑥)
1 →
lim
𝑥 Given
1 − (cos (𝑥) − sin (𝑥))
2 lim
→ 𝑥 Factor a -1 out of − cos 𝑥 + sin 𝑥
1 − cos(2𝑥)
3 lim cos(2𝑥) = cos (𝑥) − sin (𝑥)
→ 𝑥
2(1 − cos(2𝑥))
4 lim
→ 2𝑥 Multiply the numerator and denominator by 2
1 − cos(2𝑥)
5 2 lim
→ 2𝑥 Theorem 3
1 − cos(ℎ)
6 2(0) lim =0
→ ℎ

1 − cos (𝑥) + sin (𝑥)


lim = 0
7 → 𝑥 Multiply 2 by 0

Next, let’s see a function where we have multiple methods of finding the limit:

2 sin (𝑥)
lim
→ 𝑥

There are actually three ways we can find this limit:

SOLUTION A: Rewrite the numerator

No. STATEMENT REASON


2 sin (𝑥)
1 lim
→ 𝑥 Given
sin(𝑥)
2 lim 2 sin(𝑥) ⋅ Rewrite
( )
as 2 sin(𝑥) ⋅
( )
→ 𝑥
sin(𝑥)
3 2 lim sin(𝑥) ⋅ lim
→ → 𝑥 Theorems 3 and 5
( )
4 2(0)(1) lim = 1 and Theorem 8

29
Prepared by: Nathaniel M. Cabansay, BSCpE

No. STATEMENT REASON

2 sin (𝑥)
lim = 0
5 → 𝑥 Multiply 2 by 0 and 1

SOLUTION B: Double angle identity of cosine

No. STATEMENT REASON


2 sin (𝑥)
1 lim Given
→ 𝑥
1 − 1 + 2 sin (𝑥)
2 lim
→ 𝑥 Add and subtract 1 from the numerator
1 − (1 − 2 sin (𝑥))
3 lim
→ 𝑥 Factor a -1 from −1 + 2 sin 𝑥
1 − cos(2𝑥)
4 lim
→ 𝑥 cos(2𝑥) = 1 − 2 sin (𝑥)
2(1 − cos(2𝑥))
5 lim
→ 2𝑥 Multiply the numerator and denominator by 2
1 − cos(2𝑥)
6 2 lim
→ 2𝑥 Theorem 3

2(0) 1 − cos(ℎ)
7 lim =0
→ ℎ
2 sin (𝑥)
lim = 0
8 → 𝑥 Multiply 2 by 0

SOLUTION C: Power reduction formula for sine:

No. STATEMENT REASON


2 sin (𝑥)
1 lim
→ 𝑥 Given
1
2 2 [1 − cos(2𝑥)] 1
2 lim sin (𝑥) = [1 − cos(2𝑥)]
→ 𝑥 2
1 − cos(2𝑥)
3 lim Multiply 2 by 1/2
→ 𝑥
2(1 − cos(2𝑥))
4 lim
→ 2𝑥 Multiply the numerator and denominator by 2
1 − cos(2𝑥)
5 2 lim
→ 2𝑥 Theorem 3

30
Prepared by: Nathaniel M. Cabansay, BSCpE

No. STATEMENT REASON

2(0) 1 − cos(ℎ)
6 lim =0
→ ℎ
2 sin (𝑥)
lim = 0
7 → 𝑥 Multiply 2 by 0

See, we used three methods, and we got the same answer each time. This shows that in finding limits, multiple
methods are possible. Just choose what is easiest for you.

Now, let’s try another limit:

2 sin(𝑥) cos(𝑥)
lim
→ 2𝑥

We can go about this in two ways:

SOLUTION A: Rewrite the numerator

No. STATEMENT REASON


2 sin(𝑥) cos(𝑥)
1 lim
→ 2𝑥 Given
sin(𝑥)
lim cos(𝑥) ⋅ ( )
2 → 𝑥 Simplify the expression, then rewrite as cos(𝑥) ⋅

sin(𝑥)
3 lim cos(𝑥) ⋅ lim
→ → 𝑥 Theorem 5

( )
4 (1)(1) lim = 1 and Theorem 8

2 sin(𝑥) cos(𝑥)
lim = 1
5 → 2𝑥 Multiply 1 by 1

SOLUTION B: Double angle identity for sine

No. STATEMENT REASON


2 sin(𝑥) cos(𝑥)
1 lim
→ 2𝑥 Given
sin(2𝑥)
2 lim
→ 2𝑥 sin(2𝑥) = 2 sin(𝑥) cos(𝑥)

2 sin(𝑥) cos(𝑥)
lim = 1 sin(ℎ)
3 → 2𝑥 lim =1
→ ℎ

31
Prepared by: Nathaniel M. Cabansay, BSCpE

Yes, even here. We have used two methods and got the same answer each time.

Now let’s try exponential limits for a change:

4
lim 2+
→ 𝑛

No. STATEMENT REASON


4
1 lim 2+ Given
→ 𝑛
2
2 lim 2 1 + Factor a 2 out of the expression inside the parenthesis
→ 𝑛
2
3 2 lim 1+ Theorem 3
→ 𝑛

4
lim 2+ = 2𝑒 𝑟
4 → 𝑛 lim 1+ =e
→± 𝑛

Now let’s try another exponential limit:

10 −1
lim
→ 𝑥

No. STATEMENT REASON


10 −1
1 lim
→ 𝑥 Given

2(10 − 1)
2 lim
→ 2𝑥 Multiply the numerator and denominator by 2
10 − 1
3 2 lim Theorem 3
→ 2𝑥

10 −1 𝑎 −1
lim = 2 ln 10 𝑜𝑟 ln 100 lim = ln 𝑎
4 → 𝑥 → ℎ
∎ Optional: 𝑦 log 𝑥 = log 𝑥 and 10 = 100

WORD PROBLEMS INVOLVING LI MITS

Limits have applications in the real world as well. They can be applied in chemistry, engineering, and even in
business or finance.

Let’s have a chemistry example:

32
Prepared by: Nathaniel M. Cabansay, BSCpE

A glass beaker has 100 mL of pure water. 6M hydrochloric acid or hydrochloric acid containing 6 mol/L or 216 g/L,
(as 1 mole of hydrogen chloride is 36 g) is being poured into the beaker at a rate of 4 mL per second.

1. Express the concentration of hydrochloric acid 𝐶(𝑡) after 𝑡 minutes in moles per liter.
2. What is the long-term concentration of salt, or lim 𝐶(𝑡) in moles per liter?

Now, let’s start by getting the function modelled here. The molar concentration 𝑀(𝑡) in M of a solution is:

No. STATEMENT REASON


𝑡𝑜𝑡𝑎𝑙 𝑠𝑢𝑏𝑠𝑡𝑎𝑛𝑐𝑒 𝑖𝑛 𝑚𝑜𝑙𝑒𝑠
1 𝑀(𝑡) = Given
𝑡𝑜𝑡𝑎𝑙 𝑣𝑜𝑙𝑢𝑚𝑒 𝑖𝑛 𝑙𝑖𝑡𝑒𝑟𝑠

The amount of substance is 6 moles, and the substance is being


poured in at a rate of 4 mL per second or 0.004 L per second,
6 ⋅ 0.004 ⋅ 𝑡 so we are adding 6 times 0.004 moles of substance per second.
2 𝑀(𝑡) =
0.1 + 0.004𝑡 The initial amount of water is 100 mL or 0.1 L, then the volume
of the solution increases at a rate of 4 mL per second or 0.004
L per second.

0.024 ⋅ 𝑡
𝑀(𝑡) = M
0.1 + 0.004𝑡 Multiply 6 by 0.004
or Optional: Multiply the numerator and the denominator by 250
3
6𝑡
𝑀(𝑡) = M
25 + 𝑡 Don’t forget the units

Then, let’s get the long-term concentration by getting the limit as 𝑡 approaches +∞ in the function:

No. STATEMENT REASON


6𝑡
1 𝑀(𝑡) = Given
25 + 𝑡
6𝑡 To get the long-term concentration, get the limit of the
2 lim 𝑀(𝑡) = lim
→ → 25 + 𝑡 function as 𝑡 approaches +∞
6
lim 𝑀(𝑡) = lim
3 → → 25
+1 Factor out a 𝑡 from both the numerator and the denominator
𝑡
lim 6

lim 𝑀(𝑡) =
4 → 25 Theorem 6
lim
+1
𝑡→
6
5 lim 𝑀(𝑡) =
→ 0+1 lim = 0 and Theorems 1, 3 and 4a

lim 𝑀(𝑡) = 6 M
6 →
Add 0 and 1, then divide 6 by the sum. Don’t forget the units.

33
Prepared by: Nathaniel M. Cabansay, BSCpE

Well, well, well, we’re back to where we started. As we keep pouring that 6M hydrochloric acid into the beaker
(assuming it doesn’t overflow), the concentration of the solution starts to get closer and closer to the
concentration of the original substance, in this case 6M or 6 mol/L or 216 g/L.

Let’s now move on to a business and finance application of limits:

Compound interest is where the interest is compounded n times for every year t at an annual rate r. Its formula is:

𝑟
𝐴=𝑃 1+
𝑛

Using the formula above, derive the formula for continuous compounding (or where n becomes infinite).

No. STATEMENT REASON


𝑟
1 𝐴 =𝑃 1+ Given
𝑛
𝑟
2 lim 𝐴 = lim 𝑃 1 + Get the limit of both sides as 𝑛 approaches +∞
→ → 𝑛
𝑟 Hold all other variables except 𝑛 constant. Then use theorems
3 𝐴 = 𝑃 lim 1+
→ 𝑛 1 and 3

𝐴 = 𝑃e 𝑟
4 lim 1+ =e
∎ → 𝑛

Now we’ve found the formula for continuous compounding. Formulas in business math are independent of the
unit of currency used.

Now, here’s a word problem involving this:

Five separate bank transactions occurred at the same time in different parts of the world. All five involve 10,000 of
their local currency and a 5% interest rate. In India, Amir deposited his rupees in a bank that compounds daily (365
days). In the Philippines, Belle deposited her pesos in a bank that compounds monthly. In France, Clair deposited
her euros in a bank that compounds annually. In the United States, David deposited his dollars in a bank that
compounds quarterly. In Japan, Eri deposited her yen in a bank that compounds continuously.

1. Who among these five would have the highest amount in their local currency after 10 years? Round off
rupees, pesos, euros, and dollars to the nearest hundredth or 2 decimal places, but round yen to the
nearest unit or whole number.
2. If you compare the local currencies, who has the highest amount in terms of Philippine pesos? (Assume
these Forex rates: 1 rupee is 0.66 pesos, 1 euro = 57 pesos, 1 dollar= 50 pesos, 1 yen= 0.47 pesos)

No. STATEMENT REASON


𝑟
𝐴=𝑃 1+ Compound interest compounded n times a year and
1 𝑛
and compound interest compounded continuously.

34
Prepared by: Nathaniel M. Cabansay, BSCpE

No. STATEMENT REASON


𝐴 = 𝑃e

Substitute P with 10 000, as this is the principal


0.05 (irrespective of local currency used), t with 10, as this is
2 𝐴 = 10000 1 +
𝑛 over a period of 10 years, and r with 0.05 as the interest
rate is 5%

Where Amir deposited, his interest is compounded 365


( )
0.05 times in a year, so let 𝑛 = 365 to see how much he’d have
3 𝐴A = 10000 1 + ≈ 16486.65
365 at the end of 10 years. Name this value 𝐴A . He uses rupees,
so round to 2 decimal places.

Where Belle deposited, her interest is compounded 12


( )
0.05 times in a year, so let 𝑛 = 12 to see how much she’d have
4 𝐴 = 10000 1 + ≈ 16470.09
12 at the end of 10 years. Name this value 𝐴 . She uses pesos,
so round to 2 decimal places.

Where Clair deposited, her interest is compounded once a


( )
0.05 year, so let 𝑛 = 1 to see how much she’d have at the end of
5 𝐴 = 10000 1 + ≈ 16288.94
1 10 years. Name this value 𝐴 . She uses euros, so round to 2
decimal places.

Where David deposited, his interest is compounded four


( )
0.05 times a year, so let 𝑛 = 4 to see how much he’d have at the
6 𝐴 = 10000 1 + ≈ 16436.19
4 end of 10 years. Name this value 𝐴 . He uses dollars, so
round to 2 decimal places.

Where Eri deposited, her interest is compounded


𝐴 = 10000𝑒 . ( )
= 16487 continuously, so use 𝐴 = 𝑃e to see how much she’d have
7
at the end of 10 years. Name this value 𝐴 . She uses yen, so
round to the nearest whole number.

𝐴 >𝐴 >𝐴 >𝐴 >𝐴 Comparing their local currencies, it appears Eri came out
8 with the most in her local currency and Clair came out with
∎ the least in her local currency.

Since Belle’s deposit is already in Philippine pesos, her final value is 16,470.09 pesos. Converting the other four to
Philippine pesos: Amir’s final value is 10,811.19 pesos; Clair’s final value is 928,469.94 pesos; David’s final value is
821,809.73; and Eri’s final value is 7,748.99. It turns out, Clair came out with the most overall while Eri came out
with the least.

35
CHAPTER 2 CONTINUITY

MODULE 6 CONTINUITY OF A FU NCTION

CONTINUITY AT A NUMBER

Limits can also be used to see if a number is continuous at a certain number or a point.

A function 𝑓(𝑥) is continuous at a number 𝑎 if it satisfies all of these three conditions:

1. 𝑓(𝑎) exists
2. lim 𝑓(𝑥) exists

3. lim 𝑓(𝑥) = 𝑓(𝑎)

If even one of these conditions is not satisfied, the function is discontinuous at that number.

That third condition seems familiar, doesn’t it? Yes. That condition forms part of Theorem 8 that was shown back
in Module 1. To recap, here’s Theorem 8 or the Substitution Theorem: If 𝑓(𝑥) is continuous at 𝑎, then
lim 𝑓(𝑥) = 𝑓(𝑎). The converse of this statement would be: If lim 𝑓(𝑥) = 𝑓(𝑎), then 𝑓(𝑥) is continuous at 𝑎. This
→ →
is under the assumption that the first two conditions hold true.

DETERMINING CONTINUITY OF FUNCTIONS PART 1

Determine the continuity of this function…:

𝑥 − 4𝑥 + 3
𝑓(𝑥) =
𝑥−1

…at the following numbers: A.) 𝑥 = 0, B.) 𝑥 = 3, and C.) 𝑥 = 1

Start with condition A: 𝑥 = 0

No. STATEMENT REASON


𝑥 − 4𝑥 + 3
1 𝑓(𝑥) =
𝑥−1 Given
(0) − 4(0) − 3
2 𝑓(0) = Substitute 𝑥 = 0
(0) − 1
3
3 𝑓(0) = Square 0 and multiply 4 by 0
−1

4 𝑓(0) = −3 Divide 3 by -1. Condition 1 is satisfied.


𝑥 − 4𝑥 + 3
5 lim 𝑓(𝑥) = lim
→ → 𝑥−1 Using statement 1, get the limit of both sides as x approaches 0.

lim[𝑥 − 4𝑥 + 3]

6 lim 𝑓(𝑥) = Theorem 6
→ lim[𝑥 − 1]

36
Prepared by: Nathaniel M. Cabansay, BSCpE

No. STATEMENT REASON


lim 𝑥 − 4 lim 𝑥 + 3
→ →
7 lim 𝑓(𝑥) = Theorems 1, 3, and 4
→ lim 𝑥 − 1

(0) − 4(0) + 3
8 lim 𝑓(𝑥) = Theorems 2 and 7a
→ (0) − 1
3
9 lim 𝑓(𝑥) = Square 0 and multiply 4 by 0
→ −1
lim 𝑓(𝑥) = −3
10 → Divide 3 by -1. Condition 2 is satisfied.
?
lim 𝑓(𝑥) = 𝑓(0)

−3 = −3
11 Condition 3 is satisfied, so the function is continuous at 𝑥 = 0
∴ 𝑓(𝑥) is continuous at 𝑥 = 0

Now let’s try condition B: 𝑥 = 3

No. STATEMENT REASON


𝑥 − 4𝑥 + 3
1 𝑓(𝑥) =
𝑥−1 Given
(3) − 4(3) + 3
2 𝑓(3) = Substitute 𝑥 = 3
(3) − 1
0
3 𝑓(3) = Square 3 and multiply 4 by 3
−2

4 𝑓(3) = 0 Divide 0 by -2. Condition 1 is satisfied.


𝑥 − 4𝑥 + 3
5 lim 𝑓(𝑥) = lim
→ → 𝑥−1 Using statement 1, get the limit of both sides as x approaches 3.

lim[𝑥 − 4𝑥 + 3]

6 lim 𝑓(𝑥) = Theorem 6
→ lim[𝑥 − 1]

lim 𝑥 − 4 lim 𝑥 + 3
→ →
7 lim 𝑓(𝑥) = Theorems 1, 3, and 4b
→ lim 𝑥 − 1

(3) − 4(3) + 3
8 lim 𝑓(𝑥) = Theorems 2 and 7a
→ (3) − 1
0
9 lim 𝑓(𝑥) = Square 3 and multiply 4 by 3
→ −2
lim 𝑓(𝑥) = 0
10 → Divide 0 by -2. Condition 2 is satisfied.
?
lim 𝑓(𝑥) = 𝑓(3)

0=0
11 Condition 3 is satisfied, so the function is continuous at 𝑥 = 0
∴ 𝑓(𝑥) is continuous at 𝑥 = 3

37
Prepared by: Nathaniel M. Cabansay, BSCpE

Now, let’s try condition C: 𝑥 = 1

No. STATEMENT REASON


𝑥 − 4𝑥 + 3
1 𝑓(𝑥) =
𝑥−1 Given
(1) − 4(1) + 3
2 𝑓(1) = Substitute 𝑥 = 3
(1) − 1
0
3 𝑓(1) = Square 3 and multiply 4 by 3.
0

4 𝑓(1) =? Indeterminate Form. Condition 1 fails.


𝑥 − 4𝑥 + 3
5 lim 𝑓(𝑥) = lim
→ 𝑥−1→
Using statement 1, get the limit of both sides as x approaches 3.
(𝑥 − 3)(𝑥 − 1)
6 lim 𝑓(𝑥) =
→ 𝑥−1 Factor (𝑥 − 4𝑥 + 3) as (𝑥 − 3)(𝑥 − 1)
lim 𝑓(𝑥) = lim(𝑥 − 3)
7 → → Cancel an (𝑥 − 1)
lim 𝑓(𝑥) = (1) − 3
8 → Theorems 1, 2 and 4b
lim 𝑓(𝑥) = −2
9 → Subtract 3 from 1. Condition 2 is satisfied.
?
lim 𝑓(𝑥) = 𝑓(1)

4 ≠ 𝑖𝑛𝑑𝑒𝑡𝑒𝑟𝑚𝑖𝑛𝑎𝑡𝑒 Because condition 1 failed, condition 3 also fails, so the function
10
∴ 𝑓(𝑥) is not continuous at 𝑥 = 1 is not continuous at 𝑥 = 1

Let’s see an example of a piecewise function this time:

3𝑥 + 1 , 𝑥≤1
𝑓(𝑥) = at 𝑥 = 1
−4𝑥 + 3 , 𝑥>1

No. STATEMENT REASON


3𝑥 + 1 , 𝑥 ≤ 1
1 𝑓(𝑥) = at 𝑥 = 1 Given
−4𝑥 + 3 , 𝑥 > 1

𝑓(1) = 3(1) + 1 Substitute 𝑥 = 1 into 3𝑥 + 1, as this is the function defined at


2
𝑥=1

3 𝑓(1) = 4 Add 3 to 1. Condition 1 is satisfied.


lim 3𝑥 + 1

4 lim 𝑓(𝑥) = Get the one-sided limits of each part of the function
→ lim −4𝑥 + 3

3 lim 𝑥 + 1 = 3(1) + 1 = 4
→ Theorems 1, 2, 3, and 4. Since the limits aren’t equal, condition
5 −4 lim 𝑥 + 3 = −4(1) + 3 = −1
→ 2 fails.

∴ 𝑓(𝑥) is not continuous at 𝑥 = 1 Because condition 2 failed, condition 3 also fails. Therefore, the
6
∎ function is not continuous at 𝑥 = 1

38
Prepared by: Nathaniel M. Cabansay, BSCpE

Don’t let piecewise functions fool you. They could actually be continuous. Here’s another example:

8𝑥 + 10𝑥 − 12 , 𝑥 ≥ −3
𝑓(𝑥) =
−7𝑥 + 9 , 𝑥 < −3

No. STATEMENT REASON


8𝑥 + 10𝑥 − 12 , 𝑥 ≥ −3
1 𝑓(𝑥) =
−7𝑥 + 9 , 𝑥 < −3 Given

𝑓(−3) = 8(−3) + 10(−3) − 12 Substitute 𝑥 = −3 into 8𝑥 + 10𝑥 − 12, as this is the part of
2
the function defined at 𝑥 = −3

3 𝑓(−3) = 72 − 30 − 12 Square -3 then multiply the result by 8, then multiply -3 by 10

𝑓(−3) = 30 Subtract 30 from 72, then subtract 12 from the result.


4
Condition 1 is satisfied.
lim 8𝑥 + 10𝑥 − 12

5 lim 𝑓(𝑥) = Get the one-sided limits of each part of the function
→ lim −7𝑥 + 9

8 lim 𝑥 + 10 lim 𝑥 − 12
→ →
= 8(−3) + 10(−3) − 12
= 72 − 30 − 12 = 30 Theorems 1, 2, 3, 4, and 8/7a. Both one-sided limits equal, so
6
condition 2 is satisfied.
−7 lim 𝑥 + 9 = −7(−3) + 9

21 + 9 = 30
?
lim 𝑓(𝑥) = 𝑓(−3)

30 = 30 Condition 3 is satisfied, so, the function is continuous at 𝑥 =


7
−3
∴ 𝑓(𝑥) is continuous at 𝑥 = −3

CONTINUITY OF PARTICULAR FU NCTIONS

Generally, a function is continuous at every number in its domain. Some particular examples include:

1. A polynomial function is continuous at every real number, or, simply put, continuous everywhere.
2. A rational function is continuous at every number in its domain.
3. A square root is continuous at every number in its domain.

CONTINUITY OF COMPOSITIONS OF FUNCTIONS

We can then extend the conditions for continuity of functions to determine the continuity of composite functions
such as sin(𝑥 ) and e . The conditions such that 𝑓(𝑔(𝑥) is continuous are similar to continuity at a number,
and are as follows:

1. lim 𝑔(𝑥) = 𝑀

2. 𝑓(𝑀) is defined

39
Prepared by: Nathaniel M. Cabansay, BSCpE

3. lim 𝑓(𝑥) exists



4. lim 𝑓(𝑥) = 𝑓(𝑀)

That last statement looks similar to Theorem 9, doesn’t it? To recap, here’s Theorem 9: If 𝑓(𝑥) is continuous at
lim 𝑔(𝑥) = 𝑀, then lim 𝑓 𝑔(𝑥) = 𝑓 lim 𝑔(𝑥) = lim 𝑓(𝑥) = 𝑓(𝑀).
→ → → →

DETERMINING CONTINUITY OF FUNCTIONS PART 2

Let’s try determining the continuity of functions:

Given 𝑓(𝑥) = and 𝑔(𝑥) = 𝑥 , determine if 𝑓 𝑔(𝑥) is continuous at 𝑥 = 2.

There are multiple methods of doing this. Since we know that a polynomial function is continuous everywhere, we
can either start with the composition of the function, or we can start by getting the limit of the inner function.

Let’s start with the composition of the function here:

No. STATEMENT REASON


2
1 𝑓(𝑥) = , 𝑔(𝑥) = 𝑥 Given
𝑥+4
2
2 𝑓(𝑔(𝑥)) = Substitute 𝑔(𝑥) into 𝑓(𝑥)
𝑥 +4
2
3 𝑓 𝑔(2) = Substitute 𝑥 = 2 into 𝑓 𝑔(𝑥)
2 +4
2 1 Square 2 then add 4 to the result, then simplify. 𝑓 𝑔(𝑎) is
4 𝑓 𝑔(2) = =
8 4 defined.
2
5 lim 𝑓 𝑔(𝑥) = lim Get the limits of both sides as x approaches 2.
→ → 𝑥 +4
lim 2

6 lim 𝑓 𝑔(𝑥) = Theorem 6
→ lim 𝑥 + 4

2
7 lim 𝑓 𝑔(𝑥) = Theorems 1 and 8/7a
→ (2) + 4
2 1
8 lim 𝑓 𝑔(𝑥) = = Square 2, then add 4 to the result, lim 𝑓 𝑔(𝑥) exists.
→ 8 4 →

?
lim 𝑓 𝑔(𝑥) = 𝑓 𝑔(2)

1 1
= The last condition is satisfied, so, the function is continuous at
9 4 4
𝑥=2
∴ 𝑓 𝑔(𝑥) is continuous at 𝑥 = 2

40
Prepared by: Nathaniel M. Cabansay, BSCpE

Now let’s try using the limit of the inner function:

No. STATEMENT REASON


2
1 𝑓(𝑥) = , 𝑔(𝑥) = 𝑥 Given
𝑥+4
lim 𝑔(𝑥) = lim 𝑥
2 → → Get the limit of 𝑔(𝑥) as x approaches 2.
lim 𝑔(𝑥) = (2)
3 → Theorem 8/7a
lim 𝑔(𝑥) = 4
4 → Square 2
2
5 𝑓(4) = From statement 4, Substitute 𝑥 = 4 into 𝑓(𝑥)
4+4
2 1
6 𝑓(4) = = Add 4 to 4 and simplify. 𝑓(𝑀) is defined.
8 4
2
7 lim 𝑓(𝑥) = lim Get the limits of both sides as x approaches 4.
→ → 𝑥+4
lim 2

8 lim 𝑓(𝑥) = Theorem 6
→ lim(𝑥 + 4)

2
9 lim 𝑓(𝑥) = Theorems 2 and 4
→ 4+4
2 1
10 lim 𝑓(𝑥) = = Add 4 to 4 and simplify the resulting fraction.
→ 8 4
?
lim 𝑓(𝑥) = 𝑓(4)

1 1
=
4 4 The last condition is satisfied, so, the function is continuous at
11
𝑓(𝑥) is continuous at 𝑥 = 4, and 𝑥=2

𝑓 𝑔(𝑥) is continuous at 𝑥 = 2


Now let’s try a trigonometric function this time:

Given 𝑤(𝑥) = 𝑣(𝑢(𝑥)), 𝑢(𝑥) = 𝑥 + , and 𝑣(𝑥) = cos(𝑥), determine if 𝑤(𝑥) is continuous at 𝑥 = √𝜋

We’ll start by getting the limit of the inner function this time.

No. STATEMENT REASON


𝜋
𝑢(𝑥) = 𝑥 + , 𝑣(𝑥) = cos(𝑥)
1 3 Given
𝑤(𝑥) = 𝑣 𝑢(𝑥)
𝜋
2 lim 𝑢(𝑥) = lim 𝑥 + Get the limit of 𝑢(𝑥) as 𝑥 approaches √𝜋
→√ →√ 3
𝜋
3 lim 𝑢(𝑥) = √𝜋 + Theorems 1, 4 and 8/7a
→√ 3

41
Prepared by: Nathaniel M. Cabansay, BSCpE

No. STATEMENT REASON


𝜋
4 lim 𝑢(𝑥) = 𝜋 + Square √𝜋
→√ 3
4𝜋
5 lim 𝑢(𝑥) = Add 𝜋 to
→√ 3
4𝜋
𝑤 √ 𝜋 = 𝑣 𝑢 √𝜋 =𝑣
3
6 4𝜋 Substitute 𝑥 = √𝜋 into 𝑤(𝑥) = 𝑣 𝑢(𝑥) or 𝑥 = into 𝑣(𝑥).
= cos
3
4𝜋 1
7 𝑤 √𝜋 = 𝑣
3
=−
2 cos = − . 𝑤(𝑎) = 𝑣(𝑀) is defined.

lim 𝑤(𝑥) = lim 𝑣 𝑢(𝑥)


→√ →√ Get the limits of both 𝑤(𝑥) and 𝑣 𝑢(𝑥) as x approaches √𝜋
8 = lim 𝑣(𝑥) = lim cos(𝑥)
→ →
or get the limits of both 𝑣(𝑥) and cos(𝑥) as x approaches

4𝜋
lim 𝑤(𝑥) = lim 𝑣(𝑥) = cos Theorem 8
9 →√ → 3
1
lim 𝑤(𝑥) = lim 𝑣(𝑥) = − = − . lim 𝑤(𝑥) = lim 𝑣(𝑥) exists.
10 →√ → 2 cos
→ →

?
lim 𝑤(𝑥) = 𝑤 √𝜋
→√
1 1
− =− The last condition is satisfied, so, the function is continuous at
11 2 2
𝑥 = √𝜋
∴ 𝑤(𝑥) is continuous at 𝑥 = √𝜋

42
Prepared by: Nathaniel M. Cabansay, BSCpE

MODULE 7 CONTINUITY OVER AN INTERVAL

CONTINUITY OVER AN OPEN INTERVAL

Sometimes, one number is not enough to show that a function is continuous or not. A function 𝑓(𝑥) is
continuous on an open interval (𝑎, 𝑏) if it is continuous at every point on that interval.

DETERMINING CONTINUITY OVER AN INTERVAL PART 1

Determine if the function is continuous over the given interval:

𝑓(𝑥) = 𝑥 + 3, (3, 6)

Since a polynomial function is continuous everywhere, the function is continuous over the given interval.

Another example, this time with a rational function:

𝑥 − 8𝑥 + 12
𝑓(𝑥) = , (1, 4)
𝑥−2

The rational function is not continuous at 𝑥 = 2, so the function is not continuous over the given interval.

CONTINUITY OVER A CLOSED INTERVAL


We can then extend the conditions for continuity over an open interval to show that a function is continuous on a
closed interval [𝑎, 𝑏]. A function is continuous over a closed interval if it satisfies these conditions:

1. It is continuous over the open interval (a, b).


2. It is continuous from the right of 𝑎
a. 𝑓(𝑎) is defined
b. lim 𝑓(𝑥) exists

c. lim 𝑓(𝑥) = 𝑓(𝑎)

3. It is continuous from the left of 𝑏
a. 𝑓(𝑏) is defined
b. lim 𝑓(𝑥) exists

c. lim 𝑓(𝑥) = 𝑓(𝑏)

This can also apply to a half-open interval such as [𝑎, 𝑏) or (𝑎, 𝑏], but only two of the three conditions above need
to be satisfied [Specifically, conditions 1 and 2 for [𝑎, 𝑏) and conditions 1 and 3 for (𝑎, 𝑏] need to be satisfied].

DETERMINING CONTINUITY OVER AN INTERVAL PART 2


Determine the continuity of the function over the given interval:

𝑓(𝑥) = 36 − 𝑥 , [−6, 6]

The function is continuous over the open interval (-6, 6).

43
Prepared by: Nathaniel M. Cabansay, BSCpE

Now let’s test each endpoint to see if it’s continuous there as well.

No. STATEMENT REASON

1 𝑓(𝑥) = 36 − 𝑥 , [−6, 6] Given


𝑓(−6) = 36 − (−6) = 0 The function is continuous over the open interval (−6, 6), so
2
𝑓(6) = 36 − (6) = 0 we then see if 𝑓(−6) and 𝑓(6) are defined. They are.
lim 𝑓(𝑥) = 36 − (−6) = 0
→ We then get the limits of the function as x approaches -6 from
3
lim 𝑓(𝑥) = 36 − (6) = 0 the right and as x approaches 6 from the left

?
lim 𝑓(𝑥) = 𝑓(−6)

0=0
4 ? We then check if lim 𝑓(𝑥) = 𝑓(𝑎) and lim 𝑓(𝑥) = 𝑓(𝑏)
lim 𝑓(𝑥) = 𝑓(6) → →

0=0

∴ 𝑓(𝑥) is continuous over [−6, 6] Since all three conditions were satisfied, the function is
5
∎ continuous over the interval

Now, let’s try a half-open interval. Determine if the function is continuous over the given interval:

𝑥 − 3𝑥 + 2
𝑓(𝑥) = , [2, +∞)
𝑥−1

No. STATEMENT REASON

𝑥 − 3𝑥 + 2
1 𝑓(𝑥) = , [2, +∞) Given
𝑥−1

(2) − 3(2) + 2 0
= 0 The function is continuous over the interval
(2, +∞), so we
2 𝑓(2) = =
(2) − 1 1 then see if 𝑓(2) is defined.

4−6+2 0
lim 𝑓(𝑥) = = =0
→ 2−1 1 We then get the limits as x approaches 2 from the right and get
3 the limit to positive infinity (it is automatically a one-sided limit,
+∞ as mentioned in Module 3)
lim 𝑓(𝑥) = = +∞
→ 1

?
lim 𝑓(𝑥) = 𝑓(2)

0=0 We then check if lim 𝑓(𝑥) = 𝑓(𝑎). All necessary conditions
4 →
∴ 𝑓(𝑥) is continuous over [2, +∞) were satisfied.

44
Prepared by: Nathaniel M. Cabansay, BSCpE

MODULE 8 TYPES OF DISCONTINUITY

DISCONTINUITIES

In the previous two modules, we checked if a function is continuous either at a given number or over a given
interval. We also encountered some discontinuous functions along the way. These discontinuities come in two
types and five forms:

1. Removable Discontinuities- as their name says, these functions can be made continuous by redefining
the function to remove the discontinuity. In this type of discontinuity, lim 𝑓(𝑥) exists, but does not equal

𝑓(𝑎).
a. Hole/Missing Point Discontinuity- The function is not defined at a single point or has a
“hole” at that point. This commonly happens with rational functions. In cases such as these,
lim 𝑓(𝑥) exists, but 𝑓(𝑎) is an indeterminate form.

b. Isolated Point Discontinuity- The function is a piecewise function where one part of the
function has a “hole”, but some other part has defined a function/constant where the point
would be, but the value does not match the limit from both sides. Simply put, lim 𝑓(𝑥) exists,

and 𝑓(𝑎) is defined, but lim 𝑓(𝑥) ≠ 𝑓(𝑎).

2. Non-Removable Discontinuities- these functions cannot be made continuous at all. This happens
whenever limits from both sides do not exist.
a. Jump Discontinuity- In this type of discontinuity, lim 𝑓(𝑥) and lim 𝑓(𝑥) both exist, but
→ →
lim 𝑓(𝑥) ≠ lim 𝑓(𝑥), so lim 𝑓(𝑥) does not exist.
→ → →
b. Infinite Discontinuity- At least one of the limits of the function at the point 𝑎 is ±∞.
c. Oscillating Discontinuity- The function seems to oscillate between two infinite quantities, such
as with cos or sin at 𝑥 = 0.

DEALING WI TH REMOVABLE DISCONTINUITI ES

Determine the type of discontinuity in this function. If the discontinuity is removable, redefine the function to
make it continuous:

𝑥 − 6𝑥 + 11𝑥 − 6
𝑓(𝑥) =
𝑥−1

No. STATEMENT REASON


𝑥 − 6𝑥 + 11𝑥 − 6
1 𝑓(𝑥) =
𝑥−1 Given

1 − 6 + 11 − 6 0 Substituting x with 1 makes the denominator 0, so we


2 𝑓(1) = =
0 0 substitute x with 1 to see what type of discontinuity is at x = 1.

Hole discontinuity at 𝑥 = 1 Since we have an indeterminate form, we have a removable


3
discontinuity at 𝑥 = 1

45
Prepared by: Nathaniel M. Cabansay, BSCpE

No. STATEMENT REASON


𝑥 − 6𝑥 + 11𝑥 − 6 Since we are asked to redefine the function, get the limit of the
4 lim 𝑓(𝑥) = lim
→ → 𝑥−1 function as x approaches 1

By the Factor Theorem and from statement 2 above, we know


(𝑥 − 1)(𝑥 − 5𝑥 + 6) that 𝑥 − 1 is a factor of 𝑥 − 6𝑥 + 11𝑥 − 6, so factor 𝑥 − 1
5 lim 𝑓(𝑥) = lim
→ → (𝑥 − 1) from the numerator. You may also factor the resulting
quadratic.
lim 𝑓(𝑥) = lim(𝑥 − 5𝑥 + 6)
6 → → Cancel an 𝑥 − 1
lim 𝑓(𝑥) = lim 𝑥 − 5 lim 𝑥 + 6
7 → → → Theorems 1, 3, and 4
lim 𝑓(𝑥) = (1) − 5(1) + 6
8 → Theorems 2 and 8/7a.
lim 𝑓(𝑥) = 1 − 5 + 6 = −2
9 → Square 1 and multiply -5 by 1, then combine the result

𝑥 − 6𝑥 + 11𝑥 − 6
𝑓(𝑥) = ,𝑥 ≠ 1
𝑥−1
11 −2 ,𝑥 = 1 Use the limit to redefine the function to make it continuous.

Determine the type of discontinuity in this function. If the discontinuity is removable, redefine the function to
make it continuous:

|𝑥 − 2| , 𝑥≠2
𝑢(𝑥) =
1 , 𝑥=2

No. STATEMENT REASON


|𝑥 − 2| , 𝑥≠2
1 𝑢(𝑥) = Given
1 , 𝑥=2
Possible isolated point discontinuity at From the boundary point of the function, we see that there is a
2 𝑥=2 discontinuity at x = 2.

lim 𝑢(𝑥) = lim|𝑥 − 2| Since we are asked to redefine the function, get the limit of the
3 → →
function as x approaches 2

Theorems 1, 2, and 9, then subtract 2 from 2. The limit does


lim 𝑢(𝑥) = |(2) − 2| = 0
4 → not match the other part of the function. This confirms the
isolated point discontinuity.
|𝑥 − 2| , 𝑥≠2
𝑢(𝑥) =
5 0 , 𝑥=2 Use the limit to redefine the function to make it continuous.

Determine the type of discontinuity in this function:

𝑥 − 8𝑥 + 15
𝑓(𝑥) = , 𝑥≠5
𝑥−5
2 , 𝑥=5

46
Prepared by: Nathaniel M. Cabansay, BSCpE

No. STATEMENT REASON


𝑥 − 8𝑥 + 15
𝑓(𝑥) = , 𝑥≠5
1 𝑥−5 Given
2 , 𝑥=5

Possible isolated point discontinuity at From the boundary point of the function, we see that there is a
2 𝑥=5 possible discontinuity at x = 5.
𝑥 − 8𝑥 + 15 Since we are asked to redefine the function, get the limit of the
3 lim 𝑓(𝑥) = lim
→ → 𝑥−5 function as x approaches 5
0 If we directly substitute 5 into the function, we get an
4 𝑓(5) =
0 indeterminate form
(𝑥 − 3)(𝑥 − 5)
5 lim 𝑓(𝑥) = lim
→ → 𝑥−5 Factor (𝑥 − 8𝑥 + 15)
lim 𝑓(𝑥) = lim 𝑥 − 3
6 → → Cancel an 𝑥 − 5

lim 𝑓(𝑥) = 5 − 3 = 2 Theorems 1, 2, and 4. The limit equals the other part of the
7 →
function.

No discontinuity
8 The function is continuous, and there is no discontinuity.

DEALING WI TH NON-REMOVABLE DISCONTINUITIES

Determine the type of discontinuity in this function.

5 − 2𝑥 , 𝑥<1
𝑓(𝑥) =
𝑥−2 , 𝑥≥1

No. STATEMENT REASON


5 − 2𝑥 , 𝑥 < 1
1 𝑓(𝑥) = Given
𝑥−2 , 𝑥 ≥1

Possible jump discontinuity at 𝑥 = 1 From the boundary point of the function, we see that there is a
2
possible discontinuity at x = 1.
lim 𝑓(𝑥) = 5 − 2(1) = 3

3 lim 𝑓(𝑥) = (1) − 2 = −1 We get the one-sided limits to confirm if this is the case.

Jump discontinuity at 𝑥 = 1 From statement 3, the one-sided limits do not equal, confirming
4
∎ the jump discontinuity’s presence.

Determine the type of discontinuity in this function:

𝑥+4 , 𝑥<4
𝑓(𝑥) =
2𝑥 , 𝑥≥4

47
Prepared by: Nathaniel M. Cabansay, BSCpE

No. STATEMENT REASON


𝑥+4 , 𝑥 <4
1 𝑓(𝑥) = Given
2𝑥 , 𝑥≥4

Possible jump discontinuity at 𝑥 = 4 From the boundary point of the function, we see that there is a
2
possible discontinuity at x = 4.
lim 𝑓(𝑥) = (4) + 4 = 8

3 lim 𝑓(𝑥) = 2(4) = 8 We get the one-sided limits to confirm if this is the case.

No discontinuity From statement 3, the one-sided limits are equal, confirming the
4
∎ function’s continuity, or that the function has no discontinuity.
Determine the type of discontinuity in this function:

𝑥+4
𝑓(𝑥) =
𝑥−4

No. STATEMENT REASON


𝑥+4
1 𝑓(𝑥) = Given
𝑥−4

2 Possible discontinuity at 𝑥 = 4 The denominator of the function becomes 0 at x = 4.


8
3 𝑓(4) = Substitute 4 into the function. Undefined value.
0
𝑥+4
4 lim 𝑓(𝑥) = lim Get the limits of both sides as x approaches 4.
→ → 𝑥−4
lim 𝑥 + 4

5 lim 𝑓(𝑥) = Theorem 6
→ lim 𝑥 − 4

(4) + 4 8
lim 𝑓(𝑥) = =
→ (4) − 4 0
6 lim 𝑓(𝑥) = −∞ Theorems 1, 2, and 4

lim 𝑓(𝑥) = +∞

At least one limit is infinite, so the


discontinuity at 𝑥 = 4 is an infinite 1
7 discontinuity. lim = ±∞
→ 𝑥

Determine the type of discontinuity in this function:

𝜋
𝑓(𝑥) = sin
4𝑥

48
Prepared by: Nathaniel M. Cabansay, BSCpE

No. STATEMENT REASON


𝜋
1 𝑓(𝑥) = sin Given
4𝑥

2 Possible discontinuity at 𝑥 = 0 The inner function is undefined at x = 0

As this is a trigonometric or periodic function, this is a


bounded function, with the bounds at -1 and 1. However, if
3 we look at the graph, the function appears to be oscillating
wildly as we get closer and closer to x = 0. The one-sided
limits, if we tried to evaluate them, won’t exist in this case.

Oscillating discontinuity at 𝑥 = 0
4 From statement 3 above.

Let’s try another tricky limit involving a hyperbolic function.

1
𝑓(𝑥) = tanh
𝑥

No. STATEMENT REASON


1
1 𝑓(𝑥) = tanh Given
𝑥

2 Possible discontinuity at 𝑥 = 0 The inner function is discontinuous at x = 0


1
lim tanh
→ 𝑥
3 1 Get the limits as x approaches 0 from one side.
lim tanh
→ 𝑥
1
tanh lim
→ 𝑥
4 1 Theorem 9
tanh lim
→ 𝑥

49
Prepared by: Nathaniel M. Cabansay, BSCpE

No. STATEMENT REASON


tanh(−∞)
5 tanh(+∞) lim = −∞ and lim = +∞
→ →

lim tanh(𝑥)

6 lim tanh(𝑥) Theorem 9

1
lim tanh = lim tanh(𝑥) = −1
→ 𝑥 →
lim tanh(𝑥) = −1 and lim tanh(𝑥) = 1.
7 1 → →
lim tanh = lim tanh(𝑥) = 1
→ 𝑥 →

Jump discontinuity at 𝑥 = 0 From statement 7, the one-sided limits exist but do not equal,
8
∎ so the discontinuity at x = 0 is a jump discontinuity.
This example shows that jump discontinuities can happen at functions which are not piecewise functions, such as
the hyperbolic tangent above. Let’s try another example below:

|𝑥|
𝑓(𝑥) =
𝑥

No. STATEMENT REASON


|𝑥|
1 𝑓(𝑥) =
𝑥 Given

2 Possible discontinuity at 𝑥 = 0 The inner function is discontinuous at x = 0


|𝑥|
lim
→ 𝑥
3 |𝑥| Get the limits as x approaches 0 from one side.
lim
→ 𝑥

This is the graph of the function. From the left of 0, the function
4 approaches -1. From the right of 0, the function approaches 1.
The limit from both sides does not exist.

Jump discontinuity at 𝑥 = 0 From statement 7, the one-sided limits exist but do not equal,
8
∎ so the discontinuity at x = 0 is a jump discontinuity.

50
Prepared by: Nathaniel M. Cabansay, BSCpE

MODULE 9 MORE ON DISCONTINUITIES

MULTIPLE DISCONTINUITIES

A function can have discontinuities at multiple points in the function. For example, a function can have both a
removable and an infinite discontinuity, or multiple jump discontinuities. This can happen with rational functions
and other types of functions.

DEALING WI TH MULTIPLE DISCONTINUITIES


Determine the type(s) of discontinuity of this function:

𝑥 − 13𝑥 + 46𝑥 − 48
𝑓(𝑥) =
𝑥 − 11𝑥 + 24

No. STATEMENT REASON


𝑥 − 13𝑥 + 46𝑥 − 48
1 𝑓(𝑥) =
𝑥 − 11𝑥 + 24 Given

𝑥 − 13𝑥 + 46𝑥 − 48 Factor the denominator to determine where the discontinuities


2 𝑓(𝑥) =
(𝑥 − 3)(𝑥 − 8) could be.
Possible discontinuities at 𝑥 = 3 and
3 at 𝑥 = 8 Substituting x with either 3 or 8 makes the denominator 0.

𝑥 − 13𝑥 + 46𝑥 − 48
lim 𝑓(𝑥) = lim
→ → 𝑥 − 11𝑥 + 24 Get the limits from both sides as x approaches 3 and as x
4 𝑥 − 13𝑥 + 46𝑥 − 48
lim 𝑓(𝑥) = lim approaches 8.
→ → 𝑥 − 11𝑥 + 24
(𝑥 − 2)(𝑥 − 3)(𝑥 − 8)
lim 𝑓(𝑥) = lim
→ → (𝑥 − 3)(𝑥 − 8)
5 (𝑥 − 2)(𝑥 − 3)(𝑥 − 8) Factor the denominator and the numerator.
lim 𝑓(𝑥) = lim
→ → (𝑥 − 3)(𝑥 − 8)
lim 𝑓(𝑥) = lim (𝑥 − 2)
→ →
6 lim 𝑓(𝑥) = lim (𝑥 − 2) Cancel an x-3 and an x-8
→ →

lim 𝑓(𝑥) = 3 − 2 = 1

7 lim 𝑓(𝑥) = 8 − 2 = 6 Theorems 1, 2, and 4

Hole/Removable discontinuities at 𝑥 =
3 and at 𝑥 = 8 The limits from both sides exist at both x = 3 and x = 8, so the
8
function has two removable discontinuities.

Determine the type(s) of discontinuity of this function:

𝑥 − 9𝑥 + 26𝑥 − 24
𝑓(𝑥) =
𝑥 − 11𝑥 + 24

51
Prepared by: Nathaniel M. Cabansay, BSCpE

No. STATEMENT REASON


𝑥 − 9𝑥 + 26𝑥 − 24
1 𝑓(𝑥) =
𝑥 − 11𝑥 + 24 Given

𝑥 − 9𝑥 + 26𝑥 − 24 Factor the denominator to determine where the discontinuities


2 𝑓(𝑥) =
(𝑥 − 3)(𝑥 − 8) could be.
Possible discontinuities at 𝑥 = 3 and
3 at 𝑥 = 8 Substituting x with either 3 or 8 makes the denominator 0.

𝑥 − 9𝑥 + 26𝑥 − 24
lim 𝑓(𝑥) = lim
→ → 𝑥 − 11𝑥 + 24 Get the limits from both sides as x approaches 3 and as x
4 𝑥 − 9𝑥 + 26𝑥 − 24
lim 𝑓(𝑥) = lim approaches 8.
→ → 𝑥 − 11𝑥 + 24
(𝑥 − 2)(𝑥 − 3)(𝑥 − 4)
lim 𝑓(𝑥) = lim
→ → (𝑥 − 3)(𝑥 − 8)
5 (𝑥 − 2)(𝑥 − 3)(𝑥 − 4) Factor the denominator and the numerator.
lim 𝑓(𝑥) = lim
→ → (𝑥 − 3)(𝑥 − 8)
(𝑥 − 2)(𝑥 − 4)
lim 𝑓(𝑥) = lim
→ → 𝑥−8
6 (𝑥 − 2)(𝑥 − 4) Cancel an x-3
lim 𝑓(𝑥) = lim
→ → 𝑥−8
lim (𝑥 − 2)(𝑥 − 4)
lim 𝑓(𝑥) = →
→ lim (𝑥 − 8)

7 lim (𝑥 − 2)(𝑥 − 4) Theorem 6
lim 𝑓(𝑥) = →
→ lim (𝑥 − 8)

lim (𝑥 − 2) ⋅ lim (𝑥 − 4)
→ →
lim 𝑓(𝑥) =
→ lim (𝑥 − 8)

8 lim (𝑥 − 2) ⋅ lim (𝑥 − 4) Theorem 5
→ →
lim 𝑓(𝑥) =
→ lim (𝑥 − 8)

(3 − 2) ⋅ (3 − 4)
lim 𝑓(𝑥) =
→ (3 − 8)
9 (8 − 2) ⋅ (8 − 4) Theorems 1, 2, and 4
lim 𝑓(𝑥) =
→ (8 − 8)
−1 1
lim 𝑓(𝑥) = = Subtract 2, 4, and 8 from 3 and subtract 2, 4, and 8 from 8.
→ −5 5
10 24
lim 𝑓(𝑥) = = ±∞ lim = ±∞
→ 0 →

Hole discontinuity at 𝑥 = 3 and


infinite discontinuity at 𝑥 = 8 The limits from both sides exists at x = 3, while the limits are
11
infinite at x = 8.

CONTINUITY PROBLEMS

Piecewise functions don’t always need to be discontinuous, as we saw with some examples in Module 8. This only
happens if the limits of both parts of the function are equal as x approaches that boundary. Note that this can only
happen with removable discontinuities, as non-removable discontinuities cannot be redefined at all.

52
Prepared by: Nathaniel M. Cabansay, BSCpE

EXAMPLES OF C ONTINUITY PROBLEMS

Here’s an example:

Find the value 𝑘 that makes the function below continuous at 𝑥 = 2:

𝑥 −1 , 𝑥<2
𝑓(𝑥) =
𝑥+𝑘 , 𝑥≥2

No. STATEMENT REASON


𝑥 −1 , 𝑥<2
1 𝑓(𝑥) = Given
𝑥+𝑘 , 𝑥≥2

lim 𝑓(𝑥) = lim 𝑥 − 1 Get the left-hand limit of the complete part of the function as x
2 → →
approaches 2 (as this part is defined for values less than 2).
lim 𝑓(𝑥) = (2) − 1
3 → Theorems 1, 4 and 8/7a.
lim 𝑓(𝑥) = 4 − 1 = 3
4 → Square 4 and subtract 1
lim 𝑓(𝑥) = 3 For the function to be continuous, the limit from the right must

5 𝑓(2) = 3 also equal to 3, and the function must equal 3 at x = 2.

lim 𝑓(𝑥) = lim 𝑥 + 𝑘 Get the right-hand limit of the incomplete part of the function
→ →
6 𝑓(2) = 2 + 𝑘 as x approaches 2 (as this part is defined for values greater than
or equal to 2)

3 =2+𝑘 Substitute 3 into the left-hand side. Treat k as a constant, then


7
use theorems 1, 2, and 4.

8 𝑘=1 Subtract 2 from both sides

𝑥 −1 , 𝑥<2
𝑓(𝑥) =
9 𝑥+1 , 𝑥≥2 This function is continuous at x = 2.

Find the value 𝑘 that makes the function continuous at x = 7.

𝑥 − 9𝑥 + 14
𝑓(𝑥) = , 𝑥≠7
𝑥−7
𝑘 , 𝑥=7

No. STATEMENT REASON


𝑥 − 9𝑥 + 14
𝑓(𝑥) = , 𝑥≠7
1 𝑥−7 Given
𝑘 , 𝑥=7

𝑥 − 9𝑥 + 14 Get the limit of the complete part of the function as x


2 lim 𝑓(𝑥) = lim
→ → 𝑥−7 approaches 7 (as this part is defined at points other than x = 7)

53
Prepared by: Nathaniel M. Cabansay, BSCpE

No. STATEMENT REASON


(𝑥 − 2)(𝑥 − 7) Substituting 7 directly will cause an indeterminate limit, so we
3 lim 𝑓(𝑥) = lim
→ → 𝑥−7 factor the numerator.
lim 𝑓(𝑥) = lim 𝑥 − 2
4 → → Cancel an x-7
lim 𝑓(𝑥) = 7 − 2 = 5
5 → Theorems 1, 2, and 4

𝑓(7) = 5 For the function to be continuous, the function must equal 5 at


6
x = 7.

7 𝑓(7) = 𝑘 This part of the function is defined at x = 7.

8 𝑘=5 Transitive property of equality

𝑥 − 9𝑥 + 14
𝑓(𝑥) = , 𝑥≠7
𝑥−7
9 5 , 𝑥=7 This function is continuous at x = 7.

54
Prepared by: Nathaniel M. Cabansay, BSCpE

MODULE 10 THE INTERMEDIATE AND EXTREME VALU E THEOREMS

INTERMEDIATE VALUE THEOREM

One application of continuity is determining if the function passes a certain number at one point. The intermediate
value theorem states that if the function 𝑓(𝑥) is continuous on the closed interval [𝑎, 𝑏] and 𝑓(𝑎) ≠ 𝑓(𝑏), then
for every number 𝑑 between 𝑓(𝑎) and 𝑓(𝑏), there exists a number 𝑐 such that 𝑎 < 𝑐 < 𝑏 and 𝑓(𝑐) = 𝑑.

This will neither tell you the exact values of 𝑐 where the function will equal 𝑑, nor exclude a value from being
taken by a function, nor will it determine how many times the function takes on a specific value.

EXAMPLES OF THE INTERMEDIATE VALUE THEOREM


Determine if 𝑝(𝑥) = 𝑥 + 4𝑥 + 4 equals 9 somewhere in the interval [−2, 2]

No. STATEMENT REASON

1 𝑝(𝑥) = 𝑥 + 4𝑥 + 4, [−2,2] Given


𝑝(−2) = (−2) + 4(−2) + 4 A polynomial function is continuous everywhere, so test the
2 𝑝(2) = (2) + 4(2) + 4 endpoints.
𝑝(−2) = 4 − 8 + 4 = 0
3 𝑝(2) = 4 + 8 + 4 = 16 Square 2, then multiply 4 by 2 or -2 and combine the terms.

0 < 9 < 16
9 is between 0 and 16. By the intermediate value theorem, the
4 ∴ 𝑓(−2) < 𝑓(𝑐) < 𝑓(2)
function will equal 9 at a certain value 𝑐 within the interval.

Note: As the base of log 𝑥 may differ between resources, I’ll use ln 𝑥 = log 𝑥, and lg 𝑥 = log 𝑥

Determine if 𝑓(𝑥) = 𝑥 − 4 lg(4𝑥 + 2) equals 0 somewhere in the interval [0, 4]

No. STATEMENT REASON

1 𝑓(𝑥) = 𝑥 − 4 lg(4𝑥 + 2) Given


𝑓(0) = (0) − 4 lg(2) = −4 lg 2 As the logarithm only becomes 0 at 𝑥 = − , which is outside
𝑓(4) = (4) − 4 lg(18) = 16 − 4 lg 18
2 the range of the function, the function is continuous on this
interval. Test the endpoints.

−4 lg 2 < 0 < (16 − 4 lg 18) 0 is between −4 lg 2 and 16 − 4 lg 18. By the intermediate


3 ∴ 𝑓(0) < 𝑓(𝑐) < 𝑓(4) value theorem, the function will equal 0 at a certain value 𝑐
∎ within the interval.

EXTREME VALUE THEOREM

This theorem states that a function will have an absolute minimum or absolute maximum within the interval [𝑎, 𝑏].
This will be discussed in more detail in the Derivatives set of modules.

55
CHAPTER 3 MORE LIMITS

The limits in Modules 11 to 15 extend the limits found in Module 3, while Module 16 details the proof of the limits
in Module 1 and the two useful facts in Module 3.

MODULE 11 LIMITS OF EXPONENTIAL AND LOGARITHMIC FUNCTIONS

LIMITS OF EXPONENTIAL AND LOGARITHMIC FUNCTIONS

Aside from the infinite limits and limits to infinity found in Module 3, here are more useful limits involving
exponential and logarithmic functions (Let 𝑎 ∈ ℝ, meaning 𝑎 is a real number):

1
lim 𝑎 = lim = 1, 𝑎 ≠ 0 or 𝑎 ≠ ±∞
→ → 𝑎
1
lim log (𝑥) = lim log =0
→ → 𝑥

EVALU ATING LIMITS OF EXPONENTIAL AND LOGARITHMIC FUNCTIONS

Evaluate:

1
lim
→ 2

No. STATEMENT REASON


1
1 lim Given
→ 2

1
2 Theorem 9
2 →

1
3 2( ) Theorems 1, 2, and 4
1
4 2 Subtract 3 from 3

1
lim = 1
5 → 2 𝑎 = 1, 𝑎 ≠ 0 or 𝑎 ≠ ±∞

Evaluate:

lim ln(4𝑥 − 3)

No. STATEMENT REASON


lim ln(4𝑥 − 3)
1 → Given

2 ln lim(4𝑥 − 3) Theorem 9

56
Prepared by: Nathaniel M. Cabansay, BSCpE

No. STATEMENT REASON

3 ln(4 − 3) Theorems 1, 2, 3, and 4

4 ln(1) Subtract 3 from 4

lim ln(4𝑥 − 3) = 0
5 →
log 1 = 0

Evaluate:

1
lim
→ e

No. STATEMENT REASON


1
1 lim Given
→ e

1
2 Theorem 9
e →

1
3 e Theorem 2

1
lim = 1
4 → e 𝑎 = 1, 𝑎 ≠ 0 or 𝑎 ≠ ±∞

Evaluate:

lim log 𝑥

No. STATEMENT REASON


lim log (𝑥)
1 → Given

2 log lim 𝑥 Theorem 9


3 log 1 Theorem 2

lim log (𝑥) = 0


4 →
log 1 = 0

57
Prepared by: Nathaniel M. Cabansay, BSCpE

MODULE 12 LIMITS OF TRIGONOMETRIC FUNCTIONS

LIMITS OF TRIGONOMETRIC FUNCTIONS

Here, we can finally get some useful limits for sine and cosine functions and get some more useful limits for the
other four trigonometric functions. Let 𝑘 ∈ ℤ

lim sin(𝑥) = lim cos(𝑥) = 0


→( ) →

lim sin(𝑥) = lim cos(𝑥) = lim csc(𝑥) = lim sec(𝑥) = 1


→ →( ) → →( )

lim sin(𝑥) = lim cos(𝑥) = lim csc(𝑥) = lim sec(𝑥) = −1


→ →([ ] ) → →([ ] )

lim tan(𝑥) = lim cot(𝑥) = 0


→( ) →

lim tan(𝑥) = lim cot(𝑥) = 1


→ →

lim tan(𝑥) = lim cot(𝑥) = −1


→ →

Recall that the period of the sine, cosine, cosecant, and secant functions is 2π, while the period of the tangent and
cotangent functions is π.

EVALU ATING LIMITS OF TRIGONOMETRIC FUNCTIONS


Evaluate:

lim sin(2𝑥)

No. STATEMENT REASON


lim sin(2𝑥)
1 → Given

2 sin lim 2𝑥 Theorem 9


3 sin 2 lim 𝑥 Theorem 3


2 lim 𝑥 = 2𝜋
4 → Theorem 2
lim sin(𝑥)
5 → Theorem 9

lim sin(2𝑥) = 0
6 → lim sin(𝑥) = 0 where 𝑘 = 2

58
Prepared by: Nathaniel M. Cabansay, BSCpE

Evaluate:

𝜋
lim cos −𝑥
→ 2

There are two ways we can go about this: Either we use theorem 9, or we use the co-function identity
cos − 𝑥 = sin(𝑥). Let’s use Theorem 9 here.

No. STATEMENT REASON


𝜋
lim cos − 𝑥
1 → 2 Given

𝜋
2 cos lim −𝑥 Theorem 9
→ 2
𝜋
3 cos − lim 𝑥 Theorems 1 and 4
2 →

𝜋 𝜋 𝜋
− lim 𝑥 = − = 0
4 2 → 2 2 Theorem 2

lim cos(𝑥)
5 → Theorem 9
𝜋
lim cos −𝑥 = 1
6 → 2 lim cos(𝑥) = 1 where 𝑘 = 0

Let’s then try using cos − 𝑥 = sin(𝑥)

No. STATEMENT REASON


𝜋
lim cos − 𝑥
1 → 2 Given

lim sin(𝑥) 𝜋
2 → cos − 𝑥 = sin(𝑥)
2
𝜋
3 sin Theorem 8
2
𝜋
lim cos −𝑥 = 1
4 → 2 lim sin(𝑥) = 1 where 𝑘 = 0

59
Prepared by: Nathaniel M. Cabansay, BSCpE

MODULE 13 LIMITS OF INVERSE TRIGONOMETRIC FUNCTI ONS

LIMITS OF INVERSE TRIGONOMETRIC FUNCTIONS

Inverse trigonometric functions have restricted domains, unlike trigonometric functions.

𝜋
lim arcsin(𝑥) = lim arccsc(𝑥) =
→ → 2
𝜋
lim arcsin(𝑥) = lim arccsc(𝑥) = −
→ → 2

lim arccos(𝑥) = lim arcsec(𝑥) = 0


→ →

lim arccos(𝑥) = lim arcsec(𝑥) = 𝜋


→ →

𝜋
lim arctan(𝑥) = lim arccot(𝑥) =
→ → 4
𝜋
lim arctan(𝑥) = lim arccot(𝑥) = −
→ → 4

EVALU ATING LIMITS OF TRIGONOMETRIC FUNCTIONS


Evaluate:

2
lim arcsin
→ 𝑥

There are two ways we could go about this: We can do direct substitution, or we can use arcsin = arccsc(𝑥).
Let’s use direct evaluation first.

No. STATEMENT REASON


2
1 lim arcsin Given
→ 𝑥
2
2 arcsin lim Theorem 9
→ 𝑥
2 1 1
3 lim = 2 lim =2⋅ = −1 Theorems 2, 3, and 6
→ 𝑥 → 𝑥 −2
lim arcsin(𝑥)
4 → Theorem 9
2 𝜋
lim arcsin = − 𝜋
5 → 𝑥 2 lim arcsin(𝑥) = −
→ 2

Then let’s use the fact that arcsin = arccsc(𝑥).

60
Prepared by: Nathaniel M. Cabansay, BSCpE

No. STATEMENT REASON


2
1 lim arcsin Given
→ 𝑥
𝑥 1
2 lim arccsc arcsin = arccsc(𝑥)
→ 2 𝑥
𝑥 1 1
3 lim

= lim 𝑥 = ⋅ −2 = −1
2 2 → 2 Theorems 2 and 3
lim arccsc(𝑥)
4 → Theorem 9
2 𝜋
lim arcsin = − 𝜋
5 → 𝑥 2 lim arccsc(𝑥) = −
→ 2

Now let’s evaluate:

lim arctan(𝑥 + 1)

No. STATEMENT REASON


lim arctan(𝑥 + 1)
1 → Given

2 arctan lim 𝑥 + 1 Theorem 9


lim 𝑥 + 1 = 0 + 1 = 1
3 → Theorems 1, 2, and 4
lim arctan(𝑥)
4 → Theorem 9
𝜋
lim arctan(𝑥 + 1) = 𝜋
5 → 4 lim arctan(𝑥) =
→ 4

61
Prepared by: Nathaniel M. Cabansay, BSCpE

MODULE 14 LIMITS OF HYPERBOLIC FUNCTIONS

LIMITS OF HYPERBOLIC FUNCTI ONS

As described in Module 3, hyperbolic functions are special types of exponential functions.

lim sinh(𝑥) = lim csch(𝑥) = 1


→ √ → √

lim sinh(𝑥) = lim csch(𝑥) = −1


→ √ → √

lim cosh(𝑥) = lim sech(𝑥) = 1


→ →

lim tanh(𝑥) = 0

EVALU ATING LIMITS OF HYPERBOLIC FU NCTIONS


Evaluate:

lim cosh(1 − 𝑥)

No. STATEMENT REASON


lim cosh(1 − 𝑥)
1 → Given

2 cosh lim 1 − 𝑥 Theorem 9


lim 1 − 𝑥 = 1 − lim 𝑥 = 1 − 1 = 0
3 → → Theorems 1, 2, and 4
lim cosh(𝑥)
4 → Theorem 9

lim cosh(1 − 𝑥) = 1
5 → lim cosh(𝑥) = 1


Evaluate:

𝑥
lim sinh
→ √ 2

No. STATEMENT REASON


𝑥
1 lim sinh Given
→ √ 2
𝑥
2 sinh lim Theorem 9
→ √ 2
1 1
3 lim 𝑥 = ln 3 + 2√2 Theorems 2 and 3
2 → √ 2

62
Prepared by: Nathaniel M. Cabansay, BSCpE

No. STATEMENT REASON


1
𝑥 → ln 3 + 2√2
2
𝑥 → ln 3 + 2√2

4 𝑥 → ln 2 + 2√2 + 1 Simplify the logarithm first

𝑥 → ln √2 + 1

𝑥 → ln √2 + 1
lim sinh(𝑥)
5 → √ Theorem 9
𝑥
lim sinh = 1
6 → √ 2 lim sinh(𝑥) = 1
→ √

Evaluate:

lim tanh(𝑥 + 1)

No. STATEMENT REASON


lim tanh(𝑥 + 1)
1 → Given

2 tanh lim 𝑥 + 1 Theorem 9


lim 𝑥 + 1 = lim 𝑥 + 1 = −1 + 1 = 0
3 → → Theorems 1, 2, and 4
lim tanh(𝑥)
4 → Theorem 9

lim tanh(𝑥 + 1) = 0
5 → lim tanh(𝑥) = 0

63
Prepared by: Nathaniel M. Cabansay, BSCpE

MODULE 15 LIMITS OF INVERSE H YPERBOLIC FUNCTIONS

LIMITS OF INVERSE HYPERBOLIC FUNCTIONS

Inverses of exponential functions are logarithmic functions.

lim arsinh(𝑥) = lim arcsch(𝑥) = ln √2 + 1


→ →

lim arsinh(𝑥) = lim arcsch(𝑥) = ln √2 − 1


→ →

lim arcosh(𝑥) = lim arsech(𝑥) = 0


→ →

EVALU ATING LIMITS OF INVERSE HYPERBOLIC FUNCTIONS

Evaluate:

lim arsinh(𝑥 − 1)

No. STATEMENT REASON


lim arsinh(𝑥 − 1)
1 → Given

2 arsinh lim 𝑥 − 1 Theorem 9


lim 𝑥 − 1 = lim 𝑥 − 1 = 2 − 1 = 1
3 → → Theorems 1, 2, and 4
lim arsinh(𝑥)
4 → Theorem 9

lim arsinh(𝑥 − 1) = ln √2 + 1
5 → lim arsinh(𝑥) = ln √2 + 1

Evaluate:

lim arcosh(2𝑥)

No. STATEMENT REASON


lim arcosh(2𝑥)
1 → Given

2 arcosh lim 2𝑥 Theorem 9


1+
lim 2𝑥 = 2 lim 𝑥 = 2 = 1+
3 → →
2 Theorems 2 and 3

lim arcosh(𝑥)
4 → Theorem 9

lim arcosh(2𝑥) = 0
5 → lim arcosh(𝑥) = 0

64
Prepared by: Nathaniel M. Cabansay, BSCpE

MODULE 16 PROOFS OF LIMIT PROPERTIES

EPSILON-DELTA (Ε, Δ) DEFINITION OF A LIMIT

Here, we will use a more formal definition of a limit to prove the limit properties. This limit is known as the
epsilon-delta or (ε, δ) definition of a limit and is as follows:

lim 𝑓(𝑥) = 𝐿 if and only if for all 𝜀 > 0, there exists a 𝛿 > 0 such that |𝑓(𝑥) − 𝐿| < 𝜀 if 0 < |𝑥 − 𝑎| < 𝛿.

Sometimes, we are lazy to use words, so we use this symbolic definition:

lim 𝑓(𝑥) = 𝐿 ⇔ ∀ 𝜀 > 0 , ∃ 𝛿 > 0 𝑠. 𝑡. 0 < |𝑥 − 𝑎| < 𝛿 → |𝑓(𝑥) − 𝐿| < 𝜀


The symbol ⇔ means “if and only if”, the symbol ∀ means “for all”, the ∃ means “there exists”, 𝑠. 𝑡. means “such
that”, and → means “if”. Note that 0 < |𝑥 − 𝑎| < 𝛿 came first in the symbolic definition, but last in the worded
definition. This is because this is the condition to be satisfied so we could say that the limit exists.

PROVING A LIMIT USING (Ε, Δ) DEFINITION

Prove lim 𝑥 + 5 = 8 using the (ε, δ) definition of the limit.


No. STATEMENT REASON


lim 𝑥 + 5 = 8
1 → Given
|(𝑥 + 5) − 8| < 𝜀
2 0 < |𝑥 − 3| < 𝛿 From the (ε, δ) definition of the limit.

3 |(𝑥 + 5) − 8| < 𝜀 Take the first part of statement 2

4 |𝑥 − 3| < 𝜀 Subtract 8 from 5

0 < |𝑥 − 3| < 𝛿 = 𝜀 Take the second part of statement 2. From statement 4, assume
5
δ=ε

|(𝑥 + 5) − 8| = |𝑥 − 3| Assume the second part of statement 2 is true and equate to


6
statement 4.

7 |(𝑥 + 5) − 8| < 𝜀 From statement 4.

∴ lim 𝑥 + 5 = 8
8 →
From statement 2

Let’s try another example: Prove lim 𝑥 = 16 using the (ε, δ) definition of the limit

No. STATEMENT REASON


lim 𝑥 = 16
1 → Given
|𝑥 − 16| < 𝜀
2 0 < |𝑥 − 4| < 𝛿 From the (ε, δ) definition of the limit.

65
Prepared by: Nathaniel M. Cabansay, BSCpE

No. STATEMENT REASON

3 |𝑥 − 16| < 𝜀 Take the first part of statement 2

4 |(𝑥 − 4)(𝑥 + 4)| < 𝜀 Factor 𝑥 − 16 as a difference of two squares

5 |𝑥 − 4||𝑥 + 4| < 𝜀 |𝑎𝑏| = |𝑎| ⋅ |𝑏|


𝜀
6 |𝑥 − 4| < Divide both sides by |𝑥 + 4|
|𝑥 + 4|

7 0 < |𝑥 − 4| < 1 Take the second part of statement 2. Assume δ = 1

8 −1 < 𝑥 − 4 < 1 Remove absolute value bars by rewriting statement 7.

9 7 <𝑥+4 <9 Add 8 to all parts of the inequality to convert 𝑥 − 4 into 𝑥 + 4


𝜀 Assume 𝛿 = min 1, . This means we assume which of the
10 0 < |𝑥 − 4| < 𝛿 = min 1,
9
two is smaller.

|𝑥 − 16| = |𝑥 − 4||𝑥 + 4| Assume the second part of statement 2 is true and equate to
11
statement 5.

12 |𝑥 − 16| < 9|𝑥 − 4| From statement 9, we know |𝑥 + 4| < 9


𝜀
13 |𝑥 − 16| < 9 From statement 10, we know |𝑥 − 4| ≤
9

14 |𝑥 − 16| < 𝜀 Multiply by 9.

∴ lim 𝑥 = 16
15 →
From statement 2

PROVING THE BASIC LI MIT THEOREMS


Here, we will prove the ten basic limit theorems (though twelve proofs are presented here as Theorem 4 and
Theorem 7 are split into two). For the purposes of these proofs, let 𝐿 = lim 𝑓(𝑥), 𝑀 = lim 𝑔(𝑥), 𝑁 = lim ℎ(𝑥),
→ → →
assume all three limits exist and 𝑐 be a constant.

THEOREM 1: LIMIT OF A CONSTANT

Let’s start by proving the limit of a constant:

lim 𝑐 = 𝑐

No. STATEMENT REASON


lim 𝑐 = 𝑐
1 → Given
|𝑐 − 𝑐| < 𝜀
2 0 < |𝑥 − 𝑎| < 𝛿 From the (ε, δ) definition of a limit.

3 |𝑐 − 𝑐| < 𝜀 Take the first part of statement 2

66
Prepared by: Nathaniel M. Cabansay, BSCpE

No. STATEMENT REASON

4 0<𝜀 Subtract c from c.

|𝑐 − 𝑐| = 0 Assume the second part of statement 2 is true and equate to


5
statement 4

6 |𝑐 − 𝑐| < 𝜀 From statement 4.

lim 𝑐 = 𝑐
7 →
From statement 2

THEOREM 2: IDENTITY LIMIT


lim 𝑥 = 𝑎

No. STATEMENT REASON


lim 𝑥 = 𝑎
1 → Given
|𝑥 − 𝑎| < 𝜀
2 0 < |𝑥 − 𝑎| < 𝛿 From (ε, δ) definition of a limit

3 |𝑥 − 𝑎| < 𝜀 Take the first part of statement 2

4 0 < |𝑥 − 𝑎| < 𝛿 = 𝜀 Take the second part of statement 2 and set δ = ε.

|𝑥 − 𝑎| = |𝑥 − 𝑎| Assume the second part of statement 2 is true and equate to


5
statement 3

6 |𝑥 − 𝑎| < 𝛿 From statement 4

7 |𝑥 − 𝑎| < 𝜀 Since δ = ε

lim 𝑥 = 𝑎
8 →
From statement 2

THEOREM 3: CONSTANT MULTIPLE RULE


lim 𝑐𝑓(𝑥) = 𝑐𝐿

No. STATEMENT REASON


lim 𝑐𝑓(𝑥) = 𝑐𝐿
1 → Given
|𝑐𝑓(𝑥) − 𝑐𝐿| < 𝜀
2 0 < |𝑥 − 𝑎| < 𝛿 From (ε, δ) definition of a limit.

3 |𝑐𝑓(𝑥) − 𝑐𝐿| < 𝜀 Take the first part of statement 2.

4 |𝑐(𝑓(𝑥) − 𝐿)| < 𝜀 Factor 𝑐

5 |𝑐||𝑓(𝑥) − 𝐿| < 𝜀 |𝑎𝑏| = |𝑎||𝑏|

67
Prepared by: Nathaniel M. Cabansay, BSCpE

No. STATEMENT REASON


𝜀
6 |𝑓(𝑥) − 𝐿| < Divide both sides by |𝑐|
|𝑐|

|𝑐𝑓(𝑥) − 𝑐𝐿| = |𝑐||𝑓(𝑥) − 𝐿| Assume the second part of statement 2 is true and equate to
7
statement 5
𝜀
8 |𝑐𝑓(𝑥) − 𝑐𝐿| < |𝑐| From statement 6
|𝑐|

9 |𝑐𝑓(𝑥) − 𝑐𝐿| < 𝜀 Multiply 𝑐 by | |

lim 𝑐𝑓(𝑥) = 𝑐𝐿
10 →
From statement 2

THEOREM 4: LIMIT OF A SUM OR DIFFERENCE

Here, we will have to split each part of Theorem 4 by proving the limit of a sum then using that limit to prove the
limit of a difference.

THEOREM 4A: LIMIT OF A SUM

lim [𝑓(𝑥) + 𝑔(𝑥)] = 𝐿 + 𝑀


No. STATEMENT REASON


lim [𝑓(𝑥) + 𝑔(𝑥)] = 𝐿 + 𝑀
1 → Given
𝑓(𝑥) + 𝑔(𝑥) − (𝐿 + 𝑀) < 𝜀
2 0 < |𝑥 − 𝑎| < 𝛿 From the (ε, δ) definition of a limit

3 𝑓(𝑥) + 𝑔(𝑥) − (𝐿 + 𝑀) < 𝜀 Take the first part of statement 2

4 |𝑓(𝑥) + 𝑔(𝑥) − 𝐿 − 𝑀| < 𝜀 Multiply (𝐿 + 𝑀) by -1

5 |(𝑓(𝑥) − 𝐿) + (𝑔(𝑥) − 𝑀)| < 𝜀 Rearrange the terms

6 |𝑓(𝑥) − 𝐿| + |𝑔(𝑥) − 𝑀| < 𝜀 Split the sum

7 2|𝑓(𝑥) − 𝐿| < 𝜀 Set |𝑔(𝑥) − 𝑀| = |𝑓(𝑥) − 𝐿|


𝜀
8 |𝑓(𝑥) − 𝐿| < Divide both sides by 2
2
𝜀
9 |𝑔(𝑥) − 𝑀| < Set |𝑓(𝑥) − 𝐿| = |𝑔(𝑥) − 𝑀|
2

10 𝑓(𝑥) + 𝑔(𝑥) − (𝐿 + 𝑀) = |(𝑓(𝑥) − 𝐿) + (𝑔(𝑥) − 𝑀)| Assume the second part of statement 2 is true
and equate to statement 5

11 𝑓(𝑥) + 𝑔(𝑥) − (𝐿 + 𝑀) ≤ |𝑓(𝑥) − 𝐿| + |𝑔(𝑥) − 𝑀| Use the triangle inequality |𝑎 + 𝑏| ≤ |𝑎| + |𝑏|
𝜀 𝜀
12 𝑓(𝑥) + 𝑔(𝑥) − (𝐿 + 𝑀) < + From statements 8 and 9
2 2

13 𝑓(𝑥) + 𝑔(𝑥) − (𝐿 + 𝑀) < 𝜀 Add to

68
Prepared by: Nathaniel M. Cabansay, BSCpE

No. STATEMENT REASON

lim [𝑓(𝑥) + 𝑔(𝑥)] = 𝐿 + 𝑀


14 →
From statement 2

THEOREM 4B: LIMIT OF A DIFFERENCE

lim [𝑓(𝑥) − 𝑔(𝑥)] = 𝐿 − 𝑀


No. STATEMENT REASON


lim [𝑓(𝑥) − 𝑔(𝑥)] = 𝐿 − 𝑀
1 → Given
lim [𝑓(𝑥) − 𝑔(𝑥)] = lim [𝑓(𝑥) − 𝑔(𝑥)]
2 → → Reflexive Property of Equality
lim [𝑓(𝑥) − 𝑔(𝑥)] = lim [𝑓(𝑥) + (−1)𝑔(𝑥)]
3 → → Rewrite −𝑔(𝑥) as +(−1) 𝑔(𝑥)
lim [𝑓(𝑥) − 𝑔(𝑥)] = 𝐿 + (−1)𝑀 lim 𝑓(𝑥) = 𝐿, lim 𝑔(𝑥) = 𝑀 and Theorems 3 and 4a
4 →
→ →

lim [𝑓(𝑥) − 𝑔(𝑥)] = 𝐿 − 𝑀


5 →
Multiply M by -1

THEOREM 5: LIMIT OF A PRODUCT


lim [𝑓(𝑥)𝑔(𝑥)] = 𝐿𝑀

No. STATEMENT REASON


lim [𝑓(𝑥)𝑔(𝑥)] = 𝐿𝑀
1 → Given
lim 𝑓(𝑥) = 𝐿

lim [𝑓(𝑥) − 𝐿] = 0
→ Use Theorems 1 and 4b to rewrite the limits of
lim 𝑔(𝑥) = 𝑀
2 → the individual functions or simply subtract L and M
lim [𝑔(𝑥) − 𝑀] = 0 from both sides.

lim [(𝑓(𝑥) − 𝐿)(𝑔(𝑥) − 𝑀)] = 0

|(𝑓(𝑥) − 𝐿)(𝑔(𝑥) − 𝑀) − 0| < 𝜀 From the (ε, δ) definition of a limit based on


3 0 < |𝑥 − 𝑎| < 𝛿 statement 2.

4 |(𝑓(𝑥) − 𝐿)(𝑔(𝑥) − 𝑀)| < 𝜀 Take the first part of statement 3 and remove -0.

5 |𝑓(𝑥) − 𝐿||𝑔(𝑥) − 𝑀| < 𝜀 |𝑎𝑏| = |𝑎||𝑏|

6 |𝑓(𝑥) − 𝐿| < 𝜀 Set |𝑔(𝑥) − 𝑀| = |𝑓(𝑥) − 𝐿|

7 |𝑓(𝑥) − 𝐿| < √𝜀 Get the square roots of both sides

8 |𝑔(𝑥) − 𝑀| < √𝜀 Set |𝑓(𝑥) − 𝐿| = |𝑔(𝑥) − 𝑀|

|(𝑓(𝑥) − 𝐿)(𝑔(𝑥) − 𝑀) − 0| = |𝑓(𝑥) − 𝐿||𝑔(𝑥) − 𝑀| Assume the second part of statement 3 is true and
9
equate to statement 5

69
Prepared by: Nathaniel M. Cabansay, BSCpE

No. STATEMENT REASON

10 |(𝑓(𝑥) − 𝐿)(𝑔(𝑥) − 𝑀)| < √𝜀 √𝜀 From statements 7 and 8

11 |(𝑓(𝑥) − 𝐿)(𝑔(𝑥) − 𝑀)| < 𝜀 Multiply √𝜀 by √𝜀


lim [(𝑓(𝑥) − 𝐿)(𝑔(𝑥) − 𝑀)] = 0
12 → From statement 3
[𝑓(𝑥) − 𝐿][𝑔(𝑥) − 𝑀] = 𝑓(𝑥)𝑔(𝑥) − 𝑀𝑓(𝑥) − 𝐿𝑔(𝑥) + 𝐿𝑀
13 Multiply [𝑓(𝑥) − 𝐿] by [𝑔(𝑥) − 𝑀]

𝑓(𝑥)𝑔(𝑥) = [𝑓(𝑥) − 𝐿][𝑔(𝑥) − 𝑀] + 𝑀𝑓(𝑥) + 𝐿𝑔(𝑥) − 𝐿𝑀 Add 𝑀𝑓(𝑥) and 𝐿𝑔(𝑥) to both sides and subtract
14
𝐿𝑀 from both sides
lim 𝑓(𝑥)𝑔(𝑥) = lim ([𝑓(𝑥) − 𝐿][𝑔(𝑥) − 𝑀] + 𝑀𝑓(𝑥) +
15 Get the limits of both sides as 𝑥 approaches 𝑎
→ →
𝐿𝑔(𝑥) − 𝐿𝑀)
lim 𝑓(𝑥)𝑔(𝑥) = lim([𝑓(𝑥) − 𝐿][𝑔(𝑥) − 𝑀]) + lim 𝑀𝑓(𝑥) +
→ → →
16 lim 𝐿𝑔(𝑥) − lim 𝐿𝑀 Theorem 4
→ →

lim 𝑓(𝑥)𝑔(𝑥) = 0 + 𝐿𝑀 + 𝐿𝑀 − 𝐿𝑀 lim 𝑓(𝑥) = 𝐿, lim 𝑔(𝑥) = 𝑀, then use theorems 1


17 →
→ →
and 3, and statement 12

18 lim [𝑓(𝑥)𝑔(𝑥)] = 𝐿𝑀

Add LM to LM, subtract LM from 2LM.

THEOREM 6: LIMIT OF A QUOTIENT


𝑓(𝑥) 𝐿
lim = ,𝑀 ≠ 0
→ 𝑔(𝑥) 𝑀

No. STATEMENT REASON


𝑓(𝑥) 𝐿
1 lim = ,𝑀 ≠ 0 Given
→ 𝑔(𝑥) 𝑀

1 1
2 lim 𝑓(𝑥) ⋅ = 𝐿 ⋅ ,𝑀 ≠ 0 Rewrite the fraction as a product and use Theorem 5
→ 𝑔(𝑥) 𝑀

1 1
3 lim = ,𝑀 ≠ 0 Divide both sides by lim 𝑓(𝑥) = 𝐿
→ 𝑔(𝑥) 𝑀 →

1 1
− <𝜀
4 𝑔(𝑥) 𝑀 From the (ε, δ) definition of a limit based on statement 3.
0 < |𝑥 − 𝑎| < 𝛿
1 1
5 − <𝜀 Take the first part of statement 4
𝑔(𝑥) 𝑀
𝑀 − 𝑔(𝑥)
6 <𝜀 Subtract from
𝑀𝑔(𝑥) ( )

|𝑀 − 𝑔(𝑥)| 𝑎 |𝑎|
7 <𝜀 =
|𝑀𝑔(𝑥)| 𝑏 |𝑏|
|𝑔(𝑥) − 𝑀|
8 <𝜀 |𝑀 − 𝑔(𝑥)| = |𝑔(𝑥) − 𝑀|
|𝑀𝑔(𝑥)|

70
Prepared by: Nathaniel M. Cabansay, BSCpE

No. STATEMENT REASON


lim 𝑔(𝑥) = 𝑀 ⇔ ∀ 𝜀 > 0, ∃ 𝛿 >

0 𝑠. 𝑡. 0 < |𝑥 − 𝑎| <
9 𝛿 → |𝑔(𝑥) − 𝑀| < Rewrite the (ε, δ) definition
|𝑀|
2

10 |𝑀| = |𝑀 − 𝑔(𝑥) + 𝑔(𝑥)| Assume the second part of statement 4 is true and rewrite |𝑀|

11 |𝑀| ≤ |𝑀 − 𝑔(𝑥)| + |𝑔(𝑥)| Use the triangle inequality |𝑎 + 𝑏| ≤ |𝑎| + |𝑏|

12 |𝑀| ≤ |𝑔(𝑥) − 𝑀| + |𝑔(𝑥)| |𝑀 − 𝑔(𝑥)| = |𝑔(𝑥) − 𝑀|


|𝑀|
13 |𝑀| < + |𝑔(𝑥)| From statement 9
2
|𝑀| | |
14 < |𝑔(𝑥)| Subtract from both sides.
2
2 1
15 > Get the reciprocal of both sides
|𝑀| |𝑔(𝑥)|
lim 𝑔(𝑥) = 𝑀 ⟺ ∀ 𝜀 > 0, ∃ 𝛿 >

0 𝑠. 𝑡. 0 < |𝑥 − 𝑎| <
16 𝛿 → |𝑔(𝑥) − 𝑀| < Rewrite the (ε, δ) definition
|𝑀|
𝜀
2
1 1 |𝑔(𝑥) − 𝑀| Assume the second part of statement 4 is true, 𝛿 =
17 − =
𝑔(𝑥) 𝑀 |𝑀𝑔(𝑥)| min(𝛿 , 𝛿 ), and equate to statement 8.
1 1 1 1
18 − = |𝑔(𝑥) − 𝑀| Rewrite the right-hand side
𝑔(𝑥) 𝑀 |𝑀| |𝑔(𝑥)|
1 1 1 2 |𝑀|
19 −
𝑔(𝑥) 𝑀
<
|𝑀| |𝑀| 2
𝜀 From statements 15 and 16

1 1 2 |𝑀| | |
20 −
𝑔(𝑥) 𝑀
<
|𝑀| 2
𝜀 Multiply | |
by | |
and the product by 𝜀

1 1
21 lim = ,𝑀 ≠ 0 From statement 4
→ 𝑔(𝑥) 𝑀

𝑓(𝑥) 𝐿
lim = ,𝑀 ≠ 0
22 → 𝑔(𝑥) 𝑀 Use Theorem 5 to multiply both sides by lim 𝑓(𝑥) = 𝐿

THEOREM 7: LIMIT OF A POWER OR A RADICAL

These two limits are usually discussed separately in a Basic Calculus course, but they are combined here because a
radical or root can be written as an exponent or power. This fact will come in handy later on, as we first prove
the limit of the power, then use that to prove the limit of a radical.

71
Prepared by: Nathaniel M. Cabansay, BSCpE

THEOREM 7A: LIMIT OF A POWER

We’ll have to prove this one using mathematical induction

lim [𝑓(𝑥)] = 𝐿

No. STATEMENT REASON


lim [𝑓(𝑥)] = 𝐿
1 → Given
lim[𝑓(𝑥)] = lim [𝑓(𝑥)][𝑓(𝑥)]
→ →
2 =𝐿⋅𝐿 Use Theorem 5 to prove case n = 2.
=𝐿
lim [𝑓(𝑥)] = lim [𝑓(𝑥)] [𝑓(𝑥)]
→ →
3 =𝐿 ⋅𝐿 Assume statement 1 is true then prove for case n + 1.
=𝐿

lim [𝑓(𝑥)] = 𝐿
4 →
From statement 2

THEOREM 7B: LIMIT OF A RADICAL/ROOT

lim 𝑓(𝑥) = √𝐿

No. STATEMENT REASON

1 lim 𝑓(𝑥) = √𝐿 Given


2 lim 𝑓(𝑥) = lim 𝑓(𝑥) Reflexive Property of Equality


→ →

3 lim 𝑓(𝑥) = lim [𝑓(𝑥)] Rewrite the root as a rational power


→ →

4 lim 𝑓(𝑥) = 𝐿 Theorem 7a


lim 𝑓(𝑥) = √𝐿
5 → Rewrite the rational power as a root

THEOREM 8: SUBSTITUTION THEOREM


If 𝑓(𝑥) is continuous at 𝑎, then lim 𝑓(𝑥) = 𝑓(𝑎).

No. STATEMENT REASON


If 𝑓(𝑥) is continuous at 𝑎, then lim 𝑓(𝑥) = 𝑓(𝑎).
1 → Given
|𝑓(𝑥) − 𝑓(𝑎)| < 𝜀
2 0 < |𝑥 − 𝑎| < 𝛿 From the (ε, δ) definition of a limit.

72
Prepared by: Nathaniel M. Cabansay, BSCpE

No. STATEMENT REASON


|𝑥 − 𝑎| < 𝜀 Since 𝑓(𝑥) is continuous at 𝑥 = 𝑎, there must be a δ
3 0 < |𝑥 − 𝑎| < 𝛿 greater than 0 such that Theorem 2 holds

If 𝑓(𝑥) is continuous at 𝑎, then lim 𝑓(𝑥) = 𝑓(𝑎).


4 →
From Theorem 2

THEOREM 9: LIMIT OF A COMPOSITION OF FUNCTIONS

If 𝑓(𝑥) is continuous at lim 𝑔(𝑥) = 𝑀, then lim 𝑓(𝑔(𝑥)) = 𝑓 lim 𝑔(𝑥) = lim 𝑓(𝑥) = 𝑓(𝑀).
→ → → →

No. STATEMENT REASON


If 𝑓(𝑥) is continuous at lim 𝑔(𝑥) = 𝑀,

1 then lim 𝑓(𝑔(𝑥)) = 𝑓 lim 𝑔(𝑥) = lim 𝑓(𝑥) = 𝑓(𝑀). Given
→ → →

𝑓 𝑔(𝑥) − 𝑓(𝑀) < 𝜀


2 0 < |𝑥 − 𝑎| < 𝛿 From the (ε, δ) definition of a limit.

|𝑓(𝑔(𝑥)) − 𝑓(𝑀)| < 𝜀


3 0 < |𝑔(𝑥) − 𝑀| < 𝛿 Since f(x) is continuous at M
|𝑔(𝑥) − 𝑀| < 𝛿
4 0 < |𝑥 − 𝑎| < 𝛿 Since lim 𝑔(𝑥) = 𝑀

If 𝑓(𝑥) is continuous at lim 𝑔(𝑥) = 𝑀,


5 then lim 𝑓(𝑔(𝑥)) = 𝑓 lim 𝑔(𝑥) = lim 𝑓(𝑥) = 𝑓(𝑀). From statements 2 to 4
→ → →

THEOREM 10: SQUEEZE THEOREM


If 𝑓(𝑥) ≤ 𝑔(𝑥) ≤ ℎ(𝑥) and lim 𝑓(𝑥) = lim ℎ(𝑥) or 𝐿 = 𝑁, then lim 𝑔(𝑥) = lim 𝑓(𝑥) = lim ℎ(𝑥) or 𝑀 = 𝐿 = 𝑁.
→ → → → →

No. STATEMENT REASON


If 𝑓(𝑥) ≤ 𝑔(𝑥) ≤ ℎ(𝑥)
and lim 𝑓(𝑥) = lim ℎ(𝑥) or 𝐿 = 𝑁,
→ →
1 then lim 𝑔(𝑥) = lim 𝑓(𝑥) = lim ℎ(𝑥) Given
→ → →
or 𝑀 = 𝐿 = 𝑁.
|𝑓(𝑥) − 𝐿| < 𝜀
|𝑔(𝑥) − 𝑀| < 𝜀
2 |ℎ(𝑥) − 𝑁| < 𝜀 From the (ε, δ) definition of a limit.
0 < |𝑥 − 𝑎| < 𝛿
|𝑓(𝑥) − 𝐿| < 𝜀
|𝑔(𝑥) − 𝑀| < 𝜀
3 |ℎ(𝑥) − 𝐿| < 𝜀 Set N equal to L.
0 < |𝑥 − 𝑎| < 𝛿

73
Prepared by: Nathaniel M. Cabansay, BSCpE

No. STATEMENT REASON


𝐿 − 𝜀 < 𝑓(𝑥) < 𝐿 + 𝜀
4 𝑀 − 𝜀 < 𝑔(𝑥) < 𝑀 + 𝜀 Remove the absolute values on the epsilons
𝐿 − 𝜀 < ℎ(𝑥) < 𝐿 − 𝜀

5 𝐿 − 𝜀 < 𝑓(𝑥) ≤ 𝑔(𝑥) ≤ ℎ(𝑥) < 𝐿 + 𝜀 Since 𝑓(𝑥) ≤ 𝑔(𝑥) ≤ ℎ(𝑥)


𝐿 − 𝜀 < 𝑓(𝑥) < 𝐿 + 𝜀
6 𝐿 − 𝜀 < 𝑔(𝑥) < 𝐿 + 𝜀 From statement 5, we can now set M equal to L.
𝐿 − 𝜀 < ℎ(𝑥) < 𝐿 − 𝜀
|𝑓(𝑥) − 𝐿| < 𝜀
|𝑔(𝑥) − 𝐿| < 𝜀
7 |ℎ(𝑥) − 𝐿| < 𝜀 Set M equal to L in statement 3
0 < |𝑥 − 𝑎| < 𝛿

If 𝑓(𝑥) ≤ 𝑔(𝑥) ≤ ℎ(𝑥)


and lim 𝑓(𝑥) = lim ℎ(𝑥) or 𝐿 = 𝑁,
→ →
8 then lim 𝑔(𝑥) = lim 𝑓(𝑥) = lim ℎ(𝑥) Since M and N are both equal to L.
→ → →
or 𝑀 = 𝐿 = 𝑁.

EPSILON-DELTA DEFINITIONS OF OTHER KINDS OF LIMITS


Other limits can also be defined using the (ε, δ) definition:

Left-hand limit: lim 𝑓(𝑥) = 𝐿 if and only if for all 𝜀 > 0, there exists a 𝛿 > 0 such that |𝑓(𝑥) − 𝐿| < 𝜀 if − 𝛿 <

𝑥 − 𝑎 < 0 or 𝑎 − 𝛿 < 𝑥 < 𝑎 or lim 𝑓(𝑥) = 𝐿 ⟺ ∀ 𝜀 > 0, ∃ 𝛿 > 0 𝑠. 𝑡. −𝛿 < 𝑥 − 𝑎 < 0 → |𝑓(𝑥) − 𝐿| < 𝜀

Right-hand limit: lim 𝑓(𝑥) = 𝐿 if and only if for all 𝜀 > 0, there exists a 𝛿 > 0 such that |𝑓(𝑥) − 𝐿| < 𝜀 if 0 <

𝑥 − 𝑎 < 𝛿 or 𝑎 < 𝑥 < 𝑎 + 𝛿 or lim 𝑓(𝑥) = 𝐿 ⟺ ∀ 𝜀 > 0, ∃ 𝛿 > 0 𝑠. 𝑡. 0 < 𝑥 − 𝑎 < 𝛿 → |𝑓(𝑥) − 𝐿| < 𝜀

Positive infinite limit: lim 𝑓(𝑥) = +∞ if and only if for all 𝐽 > 0, there exists a 𝛿 > 0 𝑠𝑢𝑐ℎ 𝑡ℎ𝑎𝑡 𝑓(𝑥) > 𝐽 if 0 <

|𝑥 − 𝑎| < 𝛿 or lim 𝑓(𝑥) = +∞ ⟺ ∀ 𝐽 > 0, ∃ 𝛿 > 0 𝑠. 𝑡. 0 < |𝑥 − 𝑎| < 𝛿 → 𝑓(𝑥) > 𝐽

Negative infinite limit: lim 𝑓(𝑥) = −∞ if and only if for all 𝐾 < 0, there exists a 𝛿 > 0 𝑠𝑢𝑐ℎ 𝑡ℎ𝑎𝑡 𝑓(𝑥) < 𝐾 if 0 <

|𝑥 − 𝑎| < 𝛿 or lim 𝑓(𝑥) = −∞ ⟺ ∀ 𝐾 < 0, ∃ 𝛿 > 0 𝑠. 𝑡. 0 < |𝑥 − 𝑎| < 𝛿 → 𝑓(𝑥) < 𝐾

Limit to positive infinity: lim 𝑓(𝑥) = 𝐿 if and only if for all 𝜀 > 0, there exists a 𝐽 > 0 𝑠𝑢𝑐ℎ 𝑡ℎ𝑎𝑡 |𝑓(𝑥) − 𝐿| <

𝜀 if 𝑥 > 𝐽 or lim 𝑓(𝑥) = 𝐿 ⟺ ∀ 𝜀 > 0, ∃ 𝐽 > 0 𝑠. 𝑡. 𝑥 > 𝐽 → |𝑓(𝑥) − 𝐿| < 𝜀

Limit to negative infinity: lim 𝑓(𝑥) = 𝐿 if and only if for all 𝜀 > 0, there exists a 𝐾 < 0 𝑠𝑢𝑐ℎ 𝑡ℎ𝑎𝑡 |𝑓(𝑥) − 𝐿| <

𝜀 if 𝑥 < 𝐾 or lim 𝑓(𝑥) = 𝐿 ⟺ ∀ 𝜀 > 0, ∃ 𝐾 < 0 𝑠. 𝑡. 𝑥 < 𝐾 → |𝑓(𝑥) − 𝐿| < 𝜀

74
Prepared by: Nathaniel M. Cabansay, BSCpE

Infinite limit combined with limit to infinity:

Case 1: Positive infinite limit to positive infinity: lim 𝑓(𝑥) = +∞ if and only if for all 𝐾 > 0, there exists a 𝐽 >

0 𝑠𝑢𝑐ℎ 𝑡ℎ𝑎𝑡 𝑓(𝑥) > 𝐾 if 𝑥 > 𝐽 or lim 𝑓(𝑥) = +∞ ⟺ ∀ 𝐾 > 0, ∃ 𝐽 > 0 𝑠. 𝑡. 𝑥 > 𝐽 → 𝑓(𝑥) > 𝐾

Case 2: Positive infinite limit to negative infinity: lim 𝑓(𝑥) = +∞ if and only if for all 𝐾 > 0, there exists a 𝐽 <

0 𝑠𝑢𝑐ℎ 𝑡ℎ𝑎𝑡 𝑓(𝑥) > 𝐾 if 𝑥 < 𝐽 or lim 𝑓(𝑥) = +∞ ⟺ ∀ 𝐾 > 0, ∃ 𝐽 < 0 𝑠. 𝑡. 𝑥 < 𝐽 → 𝑓(𝑥) > 𝐾

Case 3: Negative infinite limit to positiveinfinity: lim 𝑓(𝑥) = −∞ if and only if for all 𝐾 < 0, there exists a 𝐽 >

0 𝑠𝑢𝑐ℎ 𝑡ℎ𝑎𝑡 𝑓(𝑥) < 𝐾 if 𝑥 > 𝐽 or lim 𝑓(𝑥) = −∞ ⟺ ∀ 𝐾 < 0, ∃ 𝐽 > 0 𝑠. 𝑡. 𝑥 > 𝐽 → 𝑓(𝑥) < 𝐾

Case 4: Negative infinite limit to negative infinity: lim 𝑓(𝑥) = −∞ if and only if for all 𝐾 < 0, there exists a 𝐽 <

0 𝑠𝑢𝑐ℎ 𝑡ℎ𝑎𝑡 𝑓(𝑥) < 𝐾 if 𝑥 < 𝐽 or lim 𝑓(𝑥) = +∞ ⟺ ∀ 𝐾 < 0, ∃ 𝐽 < 0 𝑠. 𝑡. 𝑥 < 𝐽 → 𝑓(𝑥) < 𝐾

PROVING THE USEFUL FACTS OF LIMITS TO INFINITY

Let’s split the first fact into two.

1
lim =0
→ 𝑥

No. STATEMENT REASON


1
1 lim =0 Given
→ 𝑥
1
−0 <𝜀
2 𝑥 From the (ε, δ) definition of a limit to positive infinity.
𝑥>𝐽

1
3 𝐽= Set J =
𝜀

1
4 𝑥> Since ε > 0, this root will exist. Use the fact that x > J.
𝜀
1
5 𝑥 > Raise both sides to the nth power.
𝜀
1
6 <𝜀 Get the reciprocals of both sides
𝑥
1 Since everything is positive, we can use absolute value and
7 −0 <𝜀
𝑥 subtract 0 from the left side.

1
lim =0
8 → 𝑥 From statement 2

75
Prepared by: Nathaniel M. Cabansay, BSCpE

1
lim =0
→ 𝑥

No. STATEMENT REASON


1
1 lim =0 Given
→ 𝑥
1
−0 <𝜀
2 𝑥 From the (ε, δ) definition of a limit to positive infinity.
𝑥<𝐾

1
3 𝐾=− Set K = −
𝜀

1
4 𝑥<− Since ε > 0, this root will exist. Use the fact that x < K.
𝜀

1
5 |𝑥| > Get the absolute value of both sides.
𝜀

1
6 |𝑥 | > Raise both sides to the nth power
𝜀
1
6 <𝜀 Get the reciprocals of both sides and use the fact that ε > 0.
𝑥
1 Since everything is positive, we can use absolute value and
7 −0 <𝜀
𝑥 subtract 0 from the left side.

1
lim =0
8 → 𝑥 From statement 2

Then let’s prove the second fact:

If 𝑝(𝑥) is a polynomial function 𝑎 𝑥 + 𝑎 𝑥 + ⋯ + 𝑎 𝑥 + 𝑎 , then lim 𝑝(𝑥) = lim 𝑎 𝑥 .


→± →±

Split this up into two:

lim 𝑝(𝑥) = lim 𝑎 𝑥


→ →

No. STATEMENT REASON


lim 𝑝(𝑥) = lim 𝑎 𝑥
1 → → Given
lim 𝑝(𝑥) = lim (𝑎 𝑥 + 𝑎 𝑥 +⋯+ 𝑎 𝑥 +𝑎 )
2 → → Reflexive Property of Equality
𝑎 𝑥 𝑎 𝑥 𝑎
lim 𝑝(𝑥) = lim 𝑎 𝑥 1+ + ⋯+ +
3 → → 𝑎 𝑥 𝑎 𝑥 𝑎 𝑥 Factor 𝑎 𝑥
𝑎 𝑥 𝑎 𝑥 𝑎
4 lim 𝑝(𝑥) = lim 𝑎 𝑥 ⋅ lim
→ → →
1+
𝑎 𝑥
+⋯+
𝑎 𝑥
+
𝑎 𝑥 Theorem 5. Hold lim 𝑎 𝑥 constant.

76
Prepared by: Nathaniel M. Cabansay, BSCpE

No. STATEMENT REASON


𝑎 𝑎 𝑎
lim 𝑝(𝑥) = lim 𝑎 𝑥 ⋅ lim 1+ +⋯+ +
5 → → → 𝑎 𝑥 𝑎 𝑥 𝑎 𝑥 Simplify the expressions
lim 𝑝(𝑥) = lim 𝑎 𝑥 ⋅ (1 + 0 + ⋯ + 0 + 0) Theorems 3, 4, and lim
6 → → =0

lim 𝑝(𝑥) = lim 𝑎 𝑥


7 → → Add 0 to 1, then multiply 1 by lim 𝑎 𝑥


Now the other part:

lim 𝑝(𝑥) = lim 𝑎 𝑥


→ →

No. STATEMENT REASON


lim 𝑝(𝑥) = lim 𝑎 𝑥
1 → → Given
lim 𝑝(𝑥) = lim (𝑎 𝑥 + 𝑎 𝑥 +⋯+ 𝑎 𝑥 +𝑎 )
2 → → Reflexive Property of Equality
𝑎 𝑥 𝑎 𝑥 𝑎
lim 𝑝(𝑥) = lim 𝑎 𝑥 1+ + ⋯+ +
3 → → 𝑎 𝑥 𝑎 𝑥 𝑎 𝑥 Factor 𝑎 𝑥
𝑎 𝑥 𝑎 𝑥 𝑎
4 →
lim 𝑝(𝑥) = lim 𝑎 𝑥 ⋅ lim
→ →
1+
𝑎 𝑥
+ ⋯+
𝑎 𝑥
+
𝑎 𝑥 Theorem 5. Hold lim 𝑎 𝑥 constant.

𝑎 𝑎 𝑎
lim 𝑝(𝑥) = lim 𝑎 𝑥 ⋅ lim 1+ +⋯+ +
5 → → → 𝑎 𝑥 𝑎 𝑥 𝑎 𝑥 Simplify the expressions.
lim 𝑝(𝑥) = lim 𝑎 𝑥 ⋅ (1 + 0 + ⋯ + 0 + 0) Theorems 3, 4, and lim
6 → → =0

lim 𝑝(𝑥) = lim 𝑎 𝑥


7 → → Add 0 to 1, then multiply 1 by lim 𝑎 𝑥

77
REFERENCES

Books

Mercado, J. P. & Orines, F. B. (2016). Next century mathematics 11: Basic calculus. Phoenix Publishing House, Inc.

Online Sources

Dawkins, P. (2018, May 29). Continuity. Paul’s Online Notes. https://tutorial.math.lamar.edu/Classes/CalcI/

Continuity.aspx

Dawkins, P. (2018, May 29). The Definition of the Limit. Paul’s Online Notes. https://tutorial.math.lamar.edu/Classes/

CalcI/DefnOfLimit.aspx

Dawkins, P. (2018, May 29). Limits at Infinity, Part I. Paul’s Online Notes. https://tutorial.math.lamar.edu/Classes/

CalcI/LimitsAtInfinityI.aspx

Dawkins, P. (2018, May 30). Proof of Various Limit Properties. Paul’s Online Notes. https://tutorial.math.lamar.edu/

Classes/CalcI/LimitProofs.aspx

Exponential limit of (1+1/n)^n=e. (n.d.). EMathZone. https://www.emathzone.com/tutorials/calculus/exponential-

limit-of-11nne.html

Math Easy Solutions. (2013, May 2). Proof of Squeeze Theorem [YouTube video]. https://www.youtube.com/

watch?v=vzJ7SEBLuWo

Squeeze theorem. (2020, September 7). In Wikipedia. https://en.wikipedia.org/w/index.php?title=Squeeze_theorem

&oldid=977172792
Prepared by: Nathaniel M. Cabansay, BSCpE

EXTRAS

ABOUT THIS SET OF MODULES

This set of modules deals with limits, the first set of Calculus-related topics dealt with in many Calculus courses,
be it in Engineering, Computer Science, or other courses requiring Calculus. This concept is needed in further
topics such as derivatives and integrals, which will in turn have more applications in fields such as Engineering,
Sciences, Computer Science, and other courses.

SOME QUOTES TO LIVE BY

“Give a man a fish, he will eat for a day. Teach a man to fish, he will eat every day.” – Unknown origin

“It is not of the essence of mathematics to be conversant with the ideas of number and quantity.” – George Boole

“Don’t walk behind me; I may not lead. Don’t walk in front of me; I may not follow. Just walk beside me and be my
friend.” – Albert Camus

79

You might also like